PDA

Visualizza Versione Completa : il palloncino di hubble



Morimondo
03-05-2015, 13:24
Non riesco a capire l’esempio di Hubble: un palloncino con sopra disegnati dei puntini che rappresentano le galassie gonfiandosi fa si che tutti i punti si allontanano tra loro.
Questo esempio mi suggerisce l’idea di un universo sferico quasi completamente vuoto le galassie sarebbero distribuite in un guscio sottile in espansione come la buccia di un’arancia che maturando cresce di volume.
Se cosi fosse, se la nostra galassia fosse compresa nello spessore di questa “buccia” noi dovremmo poter vedere stelle e galassie attorno a noi ma non guardando sotto i piedi o verso l’alto, in sostanza
Il punto dove vi è stato il big bang e lo spazio opposto ove si sta espandendo l’universo dovrebbero essere vuoti a meno che lo spessore di questa buccia di arancia sia oltre l’orizzone ossia maggiore di 13 miliardi di anni luce. Ma allora l’universo dovrebbe essere più grande e più vecchio di 13 miliardi di anni.

Red Hanuman
03-05-2015, 13:54
Il palloncino è solo un esempio, purtroppo incompleto per via del fatto che dobbiamo rappresentare in due dimensioni quello che avviene in tre.
L'universo in questo esempio non è rappresentato dall'intero palloncino, ma solo dalla sua superficie; abbiamo cioè compresso la terza dimensione dell'universo sulla superficie del palloncino, per motivi di visualizzazione pratica.
Quindi, non esiste nessun centro dell'universo, e l'espansione nella terza dimensione spaziale rappresenta lo scorrere del tempo, e non una dimensione spazio a tutti gli effetti.
Stando così le cose, è ovvio che noi non possiamo guardare "all'interno" o "all'esterno" del palloncino, proprio perchè così dovremmo essere in grado di guardare indietro o avanti nel tempo.
L'universo comunque non ha più di 13,7 miliardi di anni d'età, ma la sua dimensione reale attuale si stima attorno ai 93 miliardi di a.l. (vedi QUI (http://it.wikipedia.org/wiki/Universo_osservabile)).

Morimondo
04-05-2015, 01:47
Quindi, non esiste nessun centro dell'universo, e l'espansione nella terza dimensione spaziale rappresenta lo scorrere del tempo, e non una dimensione spazio a tutti gli effetti.
(vedi QUI (http://it.wikipedia.org/wiki/Universo_osservabile)).

Grazie per la risposta.
Continuo a non capire.
Per mancanza di un centro dell'universo intendi nel senso che è impossibile rintracciarlo? Se non vi fosse un centro, origine della singolarità questa singolarità allora non era tale e era ovunque?

Oppure l'espansione dell'universo è riferita al solo spazio ossia al volume che contiene le stelle e quindi coincide più o meno con l'esempio del palloncino: non sono le stelle ad allontanarsi ma lo spazio che le contiene a gonfiarsi e a trascinarsele dietro, se cosi fosse questo spazio vuoto ma in grado di trascinarsi dietro le stelle deve avere delle belle proprietà ben più importanti delle galassie che contiene, mi fa pensare un poco a Fred Hoyle.

Enrico Corsaro
04-05-2015, 10:59
Ciao, provo a risponderti.
Lasciando stare la singolarità in sè che è un concetto matematico e non fisico, quando l'Universo era ancora giovane, e quindi ancora relativamente piccolo e caldo, ha prodotto quella che chiamiamo la radiazione cosmica di fondo, un segnale elettromagnetico che spesso viene chiamato l'eco del Big Bang. Questa radiazione la vediamo oggi in tutte le direzioni in cui osserviamo. In pratica non esiste un centro (che altrimenti corrisponderebbe ad un singolo punto), ma lo spazio continua ad espandersi in tutte le direzioni, da qualunque punto tu lo osservi.
L'esempio del palloncino, o della palla, è per farti capire che se tu sei ipoteticamente vincolato sulla superficie della sfera, e se la sfera si gonfia, vedrai allontanarsi tutto in ogni direzione sulla sua superficie da qualunque punto tu ti trovi. Lo spazio dell'Universo è una sfera a 3 dimensioni e per questo non è visualizzabile dall'esterno, come nel caso di un palloncino, ma l'analogia con il palloncino è perfettamente valida.

Hai quindi questa trasformazione
Superficie in 2D del palloncino --> Spazio in 3D dell'Universo

Come giustamente dici, questo spazio che aumenta al passare del tempo ha certamente delle proprietà interessanti. Esse sono principalmente legate alla geometria dello spazio-tempo.
Abbiamo scritto un articolo in merito (e credo ce ne siano anche altri nel portale) che ti consiglio di visionare.
Uno scritto da me lo puoi trovare QUI (http://www.astronomia.com/2015/04/22/il-modello-cosmologico-standard-%CE%BBcdm-parte-i-cose-e-come-si-ricava/).

Morimondo
04-05-2015, 14:09
Come giustamente dici, questo spazio che aumenta al passare del tempo ha certamente delle proprietà interessanti. Esse sono principalmente legate alla geometria dello spazio-tempo.
Abbiamo scritto un articolo in merito (e credo ce ne siano anche altri nel portale) che ti consiglio di visionare.
Uno scritto da me lo puoi trovare QUI (http://www.astronomia.com/2015/04/22/il-modello-cosmologico-standard-%CE%BBcdm-parte-i-cose-e-come-si-ricava/).

Grazie per la risposta, ho letto il tuo articolo e avrei domande da porti ma prima devo ancora smaltirlo, non posso dire di aver capito tutto, ora però mi è chiaro l'esempio del palloncino è lo spazio che si espande e non la materia nubi, galassie stelle "esplose" e sparate tutto attorno, quindi ora è il termine "big bang" a sembrarmi inappropriato.

Ciao

Red Hanuman
04-05-2015, 14:20
Morimondo, spulcia il portale e vatti a leggere gli articoli di Enzo (un po' più vecchi). Ce ne sono alcuni fatti esattamente per quello che ti serve.... ;)

Enrico Corsaro
04-05-2015, 19:13
Grazie per la risposta, ho letto il tuo articolo e avrei domande da porti ma prima devo ancora smaltirlo, non posso dire di aver capito tutto, ora però mi è chiaro l'esempio del palloncino è lo spazio che si espande e non la materia nubi, galassie stelle "esplose" e sparate tutto attorno, quindi ora è il termine "big bang" a sembrarmi inappropriato.

Ciao

Come ti ha detto @Red Hanuman (http://www.astronomia.com/forum/member.php?u=9), c'è molto materiale nel portale e sono certo che visionandolo ti chiarirai molti dubbi. In ogni caso, per l'articolo che ti dicevo c'è il post dedicato nel forum Articoli per cui se hai domande puoi chiedere direttamente li.
Il termine Big Bang indica semplicemente un momento iniziale dell'Universo in cui esso era talmente piccolo da essere molto caldo e denso. Lo puoi immaginare sempre in riferimento al palloncino, come un palloncino con una superficie minuscola, in cui tutta l'energia che è distribuita su di essa è talmente concentrata in quel piccolo spazio da rendere la superficie estremamente calda e densa di energia. Allo stesso modo lo puoi immaginare per uno spazio in 3D.

Morimondo
07-05-2015, 02:13
...In pratica non esiste un centro (che altrimenti corrisponderebbe ad un singolo punto), ma lo spazio continua ad espandersi in tutte le direzioni, da qualunque punto tu lo osservi.
L'esempio del palloncino, o della palla, è per farti capire che se tu sei ipoteticamente vincolato sulla superficie della sfera, e se la sfera si gonfia, vedrai allontanarsi tutto in ogni direzione sulla sua superficie da qualunque punto tu ti trovi. Lo spazio dell'Universo è una sfera a 3 dimensioni e per questo non è visualizzabile dall'esterno, come nel caso di un palloncino, ma l'analogia con il palloncino è perfettamente valida.

QUI (http://www.astronomia.com/2015/04/22/il-modello-cosmologico-standard-%CE%BBcdm-parte-i-cose-e-come-si-ricava/).

Forse non ho ben chiaro il concetto di espansione: mi immaginavo l'espansione, a causa del termine bigbang, come una sorta esplosione ove la materia e quello che era prima si allontanava dalla sua origine, il centro ossia la singolarità. Ora anche se è lo spazio che contiene le galassie a espandersi questo spazio e il suo contenuto avrebbero dovrebbero allontanarsi dalla loro origine e quindi lasciarsi dietro un vuoto un spazio senza nulla. Invece se non esiste un centro non deve esistere una regione primordiale vuota, l'esempio del panettone che lievita e dei suoi canditi che si allontanano mi piace in effetti nel panettone non vi è al centro un buco.

Però non capisco una cosa, se è lo spazio che si espande e allontana tra loro galassie e stelle e lo puo fare a velocita superiori a quelle della luce come puo essere se è vuoto? Sono forse la materia e l'energia oscura a permettere la sua espansione? Se è così perchè queste possono superare la velocita della luce?

Una raffica di domande stupide: un osservatore che si trovasse proprio dove si trovava la singolarità cosa vedrebbe? Questa regione sarebbe perfettamente uguale a qualsiasi altra regione dell'universo o esisterebbe un una sorta di fossile come la radiazione di fondo di 2,7 k°,

Ringrazio Enrico per la pronta risposta e tutti quelli che hanno risposto per la pazienza, mi complimento per il sito veramente interessante e ordinato.

Red Hanuman
07-05-2015, 07:33
Forse non ho ben chiaro il concetto di espansione: mi immaginavo l'espansione, a causa del termine bigbang, come una sorta esplosione ove la materia e quello che era prima si allontanava dalla sua origine, il centro ossia la singolarità. Ora anche se è lo spazio che contiene le galassie a espandersi questo spazio e il suo contenuto avrebbero dovrebbero allontanarsi dalla loro origine e quindi lasciarsi dietro un vuoto un spazio senza nulla. Invece se non esiste un centro non deve esistere una regione primordiale vuota, l'esempio del panettone che lievita e dei suoi canditi che si allontanano mi piace in effetti nel panettone non vi è al centro un buco.

Però non capisco una cosa, se è lo spazio che si espande e allontana tra loro galassie e stelle e lo puo fare a velocita superiori a quelle della luce come puo essere se è vuoto? Sono forse la materia e l'energia oscura a permettere la sua espansione? Se è così perchè queste possono superare la velocita della luce?

Una cosa è la velocità della luce NELLO spazio, altra cosa è la velocità di espansione DELLO spazio. Nello spazio nulla si può muovere più veloce della luce, ma lo spazio si può tranquillamente creare ex - novo in maniera tale che lo spazio creato nell'unità di tempo sia maggiore di quello che la luce può percorrere nel medesimo tempo. Le galassie, quindi, possono "allontanarsi" l'una dall'altra più velocemente della luce, proprio perchè si crea tanto spazio tra di loro, anche se di fatto possono essere "ferme".
Quello che provoca l'espansione dell'universo dev'essere proprio quell'energia, che è "oscura" proprio nel senso che non sappiamo cosa sia...


Una raffica di domande stupide: un osservatore che si trovasse proprio dove si trovava la singolarità cosa vedrebbe? Questa regione sarebbe perfettamente uguale a qualsiasi altra regione dell'universo o esisterebbe un una sorta di fossile come la radiazione di fondo di 2,7 k°,

Ringrazio Enrico per la pronta risposta e tutti quelli che hanno risposto per la pazienza, mi complimento per il sito veramente interessante e ordinato.


Il punto è che prima dell'universo non c'era nulla, nel senso che non esisteva nulla.
Quando nasce, l'universo è già completo, solo con un volume minore di quello attuale. E siccome al di fuori di esso non c'era e non c'è nulla, non esiste un riferimento per poter parlare di "centro" dell'universo, perchè l'universo appena nato è già "tutto". Quindi, la singolarità è dovunque intorno a noi, ed ecco perchè la CMB la puoi trovare ovunque nello spazio.... ;)

Morimondo
07-05-2015, 14:58
ULe galassie, quindi, possono "allontanarsi" l'una dall'altra più velocemente della luce, proprio perchè si crea tanto spazio tra di loro, anche se di fatto possono essere "ferme".
Quello che provoca l'espansione dell'universo dev'essere proprio quell'energia, che è "oscura" proprio nel senso che non sappiamo cosa sia...

Quindi lo spazio non si dilata o espande ma si crea ex novo (con lo zampino di energia e materia oscure immagino perchè il nulla non puo fare nulla presumo) a questo punto mi viene in mente l'universo stazionario di Fred Hoyle che ormai di fronte all'evidenza dell'espansione dell'universo pur di non ammettere il big bang ipotizzò che si creasse materia spontanea nello spazio interstellare e che questa fosse la causa dellespansione.

Qui invece si parla della creazione di spazio nello spazio (con il probabile intervento di queste benedette materia e energia oscura), ma vi è, a mio avviso, un parallelismo con la teoria del citato Fred Hoyle.

Termino con due domande, questa due cose oscure potrebbero essere loro causa della radiazione fossile?

Da quanto tempo si parla di espansione dello spazio al posto di espansione limitata alla materia che lo occupa, per me è cosa assolutamente nuova (e difficile da digerire).

Grazie

DarknessLight
07-05-2015, 17:00
Morimondo Solo l energia oscura è la causa dell espansione dello spazio-tempo. La materia oscura invece è materia, perciò genera gravità e di conseguenza frena l espansione, un po come la materia ordinaria (barionica).
Non sono la stessa cosa!!

Esse comunque non sono la causa della cmb (o radiazione fossile) poiché la cmb è semplicemente il resto fossile di un evento ancestrale avvenuto 380 000 anni dopo il Big bang ed in particolare rappresenta il disaccoppiamento tra materia e radiazione: significa che ad un certo punto la densità media dell universo si è abbassata a tal punto (a causa dell espansione inerziale del Big bang e dell espansione causata dalla energia oscura) da permettere che materia e radiazione si staccassero seguendo destini separati: l eco di questi fotoni emessi sono la cmb (che comprende comunque lievi fluttuazioni quantistiche di pochi centesimi di kelvin).

Per quanto riguarda l espansione, negli anni trenta hubble afferma che esiste una relazione lineare tra il Red shift della luce emessa dalle galassie e la loro distanza.
Il Red shift è lo spostamento verso lunghezze d onda maggiori di una determinata onda elettromagnetica. Avviene quando il corpo che emette l onda si sta allontanando dall osservatore. Ciò implica quindi che le galassie si stanno allontanando dalla terra (in realtà le une si allontanano dalle altre perché la terra NON è il centro dell universo: altrimenti detto PRINCIPIO COSMOLOGICO).
Questa è una conferma alle soluzioni delle equazioni di Einstein se si postula un universo OMOGENEO, ISOTROPO ed IN ESPANSIONE.
Il parametro di hubble indica la velocità di espansione dell universo. Esso varia nel corso delle ere cosmologiche.

Spero di avere aiutato.

Morimondo
07-05-2015, 17:28
Morimondo Solo l energia oscura è la causa dell espansione dello spazio-tempo. La materia oscura invece è materia, perciò genera gravità e di conseguenza frena l espansione, un po come la materia ordinaria (barionica).
Non sono la stessa cosa!!

Grazie per il chiarimento, in effetti mi chiedevo perchè la materia oscura portasse all'espansione dello spazio e magari viaggiasse oltre la velocità della luce.

Red Hanuman
07-05-2015, 22:10
Mi sembra che DarknessLight abbia completato bene le mie risposte. Comunque...


Quindi lo spazio non si dilata o espande ma si crea ex novo (con lo zampino di energia e materia oscure immagino perchè il nulla non puo fare nulla presumo) a questo punto mi viene in mente l'universo stazionario di Fred Hoyle che ormai di fronte all'evidenza dell'espansione dell'universo pur di non ammettere il big bang ipotizzò che si creasse materia spontanea nello spazio interstellare e che questa fosse la causa dellespansione.

Veramente, Hoyle fu costretto ad ipotizzare la continua formazione di materia a seguito della scoperta dell'espansione dell'universo per mantenere la sua posizione iniziale (e non come causa dell'espansione, visto che la massa tende a chiudere l'universo), e cioè per mantenere il principio cosmologico perfetto (http://it.wikipedia.org/wiki/Principio_cosmologico_perfetto).


Qui invece si parla della creazione di spazio nello spazio (con il probabile intervento di queste benedette materia e energia oscura), ma vi è, a mio avviso, un parallelismo con la teoria del citato Fred Hoyle.

Come già detto, la materia oscura non serve per spiegare l'espansione dell'universo, ma per tener conto dei movimenti dentro e tra le galassie. E' solo un'ipotesi, non un dato, infatti ci sono diverse teorie ugualmente valide fino a prova contraria. La materia oscura è solo più in linea con la teoria più in voga, la ΛCDM (http://it.wikipedia.org/wiki/Modello_Lambda-CDM), delle altre spiegazioni....


Termino con due domande, questa due cose oscure potrebbero essere loro causa della radiazione fossile?


Da quanto tempo si parla di espansione dello spazio al posto di espansione limitata alla materia che lo occupa, per me è cosa assolutamente nuova (e difficile da digerire).


Grazie

No, come già detto la radiazione fossile è un residuo del disaccoppiamento tra materia e radiazione, a sua volta conseguenza necessaria della ΛCDM...;)

Morimondo
08-05-2015, 10:27
Ringrazio tutti quelli che mi hanno risposto, ho le idee un po' più chiare, devo dire che pur avendo letto negli anni 90 qualche testo di coswmologia, naturalmente divulgativo, ed essendo abbonato a LE SCIENZE molte cose non le avevo capite.

Non ho capito dove vi è il comando per ringraziare, è forse la stellina di reputazione?

Ripeto i complimenti per il sito decisamente ricco e vario mi piacerebbe ne esistesse uno fatto così sulle scienze della terra (geologia-paleontologia-paleantropologia)

Saluti

etruscastro
08-05-2015, 13:02
Ripeto i complimenti per il sito decisamente ricco e vario mi piacerebbe ne esistesse uno fatto così sulle scienze della terra (geologia-paleontologia-paleantropologia)
potrebbe essere una buona idea per un'ulteriore sezione scientifica qua su Astronomia.com! :sneaky:

Enrico Corsaro
09-05-2015, 00:45
Non ho capito dove vi è il comando per ringraziare, è forse la stellina di reputazione?



No ma è proprio alla sinistra della stellina, dove compare scritto Grazie ;).


@Red Hanuman (http://www.astronomia.com/forum/member.php?u=9) e @DarknessLight (http://www.astronomia.com/forum/member.php?u=3442) ti han già risposto a tutto!
Una precisazione però che mi sembra doverosa per l'espansione dell'Universo e per cui bisogna prestare attenzione. In realtà l'energia oscura non è responsabile dell'espansione, ma del fatto che l'espansione (già di per se presente a prescindere) è ad oggi accelerata, cioè il tasso con cui l'Universo si espande aumenta al passare del tempo.
L'espansione in sè infatti è stata generata invece dall'esplosione iniziale (Big Bang) a causa della elevatissima densità di energia e temperatura presenti nel piccolissimo Universo primordiale.

Il tasso di espansione nel corso del tempo è regolato quindi dal bilancio energetico dei componenti (vedi l'articolo che ti ho suggerito inizialmente pubblicato qui sul portale), in particolar modo da bilancio tra materia (che tende a rallentarla) ed energia oscura (che invece tende ad accelerarla). Tutto questo sempre secondo il modello cosmologico standard.

Morimondo
11-05-2015, 02:24
No ma è proprio alla sinistra della stellina, dove compare scritto Grazie ;).
Una precisazione però che mi sembra doverosa per l'espansione dell'Universo e per cui bisogna prestare attenzione. In realtà l'energia oscura non è responsabile dell'espansione, ma del fatto che l'espansione (già di per se presente a prescindere) è ad oggi accelerata, cioè il tasso con cui l'Universo si espande aumenta al passare del tempo.
L'espansione in sè infatti è stata generata invece dall'esplosione iniziale (Big Bang) a causa della elevatissima densità di energia e temperatura presenti nel piccolissimo Universo primordiale.

Non so come non vedevo il grazie.

Enrico mi hai fatto ripiombare nel mare dei miei dubbi.;)
Il termine esplosione mi fa pensare alle immagini della Crab Nebula o della Nebulosa della Lira: la materia dei due oggetti si sta allontanando da un centro, la supernova del 1054 nella caso della Crab Nebula, e noi vediamo infatti un vuoto al centro, quindi perchè non è cosi anche per il big bang?
Da questa singolarità continua a uscire materia che rimpiazza il vuoto oppure noi, (ma perchè proprio noi) siamo oltre un orizzonte e non possiamo vedere questo centro perchè ormai fuori a causa dell'espansione accelerata o dell'inflazione?

Enrico Corsaro
11-05-2015, 05:49
Il termine esplosione mi fa pensare alle immagini della Crab Nebula o della Nebulosa della Lira: la materia dei due oggetti si sta allontanando da un centro, la supernova del 1054 nella caso della Crab Nebula, e noi vediamo infatti un vuoto al centro, quindi perchè non è cosi anche per il big bang?
Da questa singolarità continua a uscire materia che rimpiazza il vuoto oppure noi, (ma perchè proprio noi) siamo oltre un orizzonte e non possiamo vedere questo centro perchè ormai fuori a causa dell'espansione accelerata o dell'inflazione?

Il problema è proprio pensare all'esplosione iniziale come a quella di una supernova...non conviene fare questo paragone e può indurre in errore. La supernova è un oggetto dentro uno spazio in 3 dimensioni, quindi se lo vedi lo puoi localizzare in un punto ben preciso dello spazio. Il Big Bang invece riguarda lo stesso spazio (meglio dire lo spazio-tempo) e non puoi vederlo dal di fuori sostanzialmente, perchè non esiste alcun riferimento fuori dal Big Bang. Il Big Bang cioè non è localizzato dentro uno spazio, non c'è un riferimento di centro quindi, ma esso rappresenta lo spazio stesso in un primissimo istante di vita dell'Universo. Devi immaginare la cosa come uno spazio che si ingrandisce in tutte le direzioni, facendo si che ogni punto al suo interno si allontani da qualsiasi altro punto in ogni direzione tu possa guardare. Non c'è dunque un centro, perchè se ci fosse, vedresti tutto allontanarsi rispetto ad un solo punto, cosa che non avviene assolutamente. Lo spazio si espande in tutte le direzioni, da qualsiasi punto tu lo possa osservare.
L'esempio più chiaro e semplice per capirlo è sempre quello del palloncino ma non il palloncino in sè, ma ciò che avviene sulla superficie del palloncino. Mentre il palloncino gonfia, tutti i punti sulla superficie si allontanano l'uno dall'altro. Dimentica del centro del palloncino e del suo interno pieno di aria, ma pensa semplicemente che il tuo spazio-tempo è tutto rappresentato dalla superficie del palloncino, in 2 dimensioni quindi. Proietta poi la cosa in 3 dimensioni, e avrai ciò che succede all'Universo! ;)

Arzak
15-05-2015, 20:06
Avrei anch'io un mare di domande, ma mi trattengo per non abusare troppo della cortesia degli amici. Ma anche perchè, prima di porre domande troppo complesse, sento a monte il bisogno di documentarmi meglio per non sprecare il tempo altrui e per non fare figure da babbeo.
Due domandine su cui però mi azzardo a fare la figura di cui sopra non riesco a trattenerle, e quindi eccole:

- il giorno in cui verrà costruito un telescopio capace di vedere oltre i 14 miliardi di anni luce, sarò in grado di vedere il Big Bang e ciò che lo precedeva, o la domanda non ha senso?
- se delle galassie si allontanano fra loro a velocità superiori a c, immagino che fra loro non riusciranno a vedersi. Ce ne sono fra queste alcune che si allontanano dalla nostra Galassia a quella velocità? E se sì, come facciamo a vederle o a rilevare la loro esistenza?
Grazie in anticipo della pazienza.

davide1334
15-05-2015, 20:16
sulla prima direi che non si tratta di deficit tecnologico. oggetti ed eventi più lontani di tredici e rotti miliardi di anni non se ne vedono semplicemente perchè non ce ne sono,o meglio ce ne sono per via dell'espansione ma non li vedremo mai

Red Hanuman
15-05-2015, 20:28
Avrei anch'io un mare di domande, ma mi trattengo per non abusare troppo della cortesia degli amici. Ma anche perchè, prima di porre domande troppo complesse, sento a monte il bisogno di documentarmi meglio per non sprecare il tempo altrui e per non fare figure da babbeo.
Due domandine su cui però mi azzardo a fare la figura di cui sopra non riesco a trattenerle, e quindi eccole:

- il giorno in cui verrà costruito un telescopio capace di vedere oltre i 14 miliardi di anni luce, sarò in grado di vedere il Big Bang e ciò che lo precedeva, o la domanda non ha senso?

Sorry, domanda senza senso.... Visto che tutto è cominciato col BB, luce compresa, non si può vedere oltre. Anche perchè il "prima" non esiste....;)


- se delle galassie si allontanano fra loro a velocità superiori a c, immagino che fra loro non riusciranno a vedersi. Ce ne sono fra queste alcune che si allontanano dalla nostra Galassia a quella velocità? E se sì, come facciamo a vederle o a rilevare la loro esistenza?
Grazie in anticipo della pazienza.

Ce ne sono, eccome. Molte di quelle più lontane che oggi vediamo sono già oltre la cosiddetta "sfera di Hubble", che delimita la parte di universo che possiamo vedere. Oggi, vediamo la luce che è partita da loro miliardi di anni fa, ma sono destinate pian piano a scomparire dalla nostra vista. La luce che parte da loro adesso, dunque, non ci potrà mai raggiungere, e mano a mano tutte le galassie lontane scompariranno dalla nostra vista, tranne quelle legate a noi gravitazionalmente. Finché la gravità regge....:whistling:

DarknessLight
15-05-2015, 20:41
il giorno in cui verrà costruito un telescopio capace di vedere oltre i 14 miliardi di anni luce, sarò in grado di vedere il Big Bang e ciò che lo precedeva, o la domanda non ha senso?

Come ti hanno detto Davide e red vedere oltre 13,7 miliardi di anni fa è impossibile proprio perché prima di questo tempo non c era nulla. Gli oggetti più lontani che osserviamo si collocano all incirca intorno a questo limite (tipo i quasar). L universo osservabile si chiama sfera di hubble, però l universo si estende ben oltre la sfera di hubble poiché esiste un espansione accelerata che fa espandere lo spazio ad una velocità sempre maggiore e più un corpo si trova lontano da noi più l effetto di questa espansione accelerata è evidente nel senso che gli oggetti oltre la sfera di hubble si allontanano da noi a velocità superiori a quelle della luce (o meglio non sono gli oggetti ad allontanarsi ma è lo spazio ad espandersi a velocità superiori a quella della luce) perciò sono scomparsi per sempre alla nostra visuale e mai li vedremo. Ma anche gli oggetti che si collocano oltre la sfera di hubble sono comunque più giovani di 13,7 miliardi di anni poiché appunto prima non c era nulla. Si stima che l universo si estenda oltre la sfera di hubble per circa altri 90 miliardi di anni luce (mi pare ma non ne sono sicuro, forse anche molto di più). Questo secondo il modello lambda cmb.
però si può osservare l eco del Big bang ovvero la radiazione cosmica di fondo a microonde (detta anche cmb) che rappresenta il disaccoppiamento tra materia ed energia avvenuta pochi migliaia di anni dopo il Big bang. ;)


se delle galassie si allontanano fra loro a velocità superiori a c, immagino che fra loro non riusciranno a vedersi. Ce ne sono fra queste alcune che si allontanano dalla nostra Galassia a quella velocità? E se sì, come facciamo a vederle o a rilevare la loro esistenza?.

come ti ho detto prima la maggior parte dell universo si trova oltre la sfera di hubble perciò si allontana da noi a velocità superiori a quelle della luce. Perciò a noi resteranno sempre invisibili. Per ogni punto nell universo si può cmq individuare una sfera di hubble (vedi anche principio cosmologico) oltre cui i corpi non sono più osservabili. Ma tutto nell universo è comunque più giovane di 13,7 miliardi di anni. ;)

Marcos64
15-05-2015, 23:01
Corsaro, che ringrazio, esorta ad esprimersi anche i non addetti ai lavori, quindi provvedo non curandomi
della figuraccia :)
Una cosa non mi torna, ho sempre considerato la velocita' della luce una costante, in nessun modo
oltrepassabile. Da quanto leggo qui, una parte di universo non e' visibile perche' si sta allontanando a
velocita' superiori a quella della luce. Allora mi son detto, in un universo in espansione, potrebbe essere che
una galassia si allontani dalla nostra a velocita' prossime a c, e noi da lei a velocita' analoga. La velocita' reciproca sara' quindi ben superiore a quella della luce, e' cosi' che la devo intendere?

Arzak
15-05-2015, 23:21
Corsaro, che ringrazio, esorta ad esprimersi anche i non addetti ai lavori, quindi provvedo non curandomi
della figuraccia :)
Una cosa non mi torna, ho sempre considerato la velocita' della luce una costante, in nessun modo
oltrepassabile. Da quanto leggo qui, una parte di universo non e' visibile perche' si sta allontanando a
velocita' superiori a quella della luce. Allora mi son detto, in un universo in espansione, potrebbe essere che
una galassia si allontani dalla nostra a velocita' prossime a c, e noi da lei a velocita' analoga. La velocita' reciproca sara' quindi ben superiore a quella della luce, e' cosi' che la devo intendere?
Vediamo se fra non addetti ci si riesce ad aiutare. Alla peggio qualcuno ci tirerà le orecchie...
Dunque, la velocità della luce non è una costante, perchè dipende dalle caratteristiche elettromagnetiche del mezzo in cui si propaga. Se per c intendiamo la velocità della luce nel vuoto, allora sì, ed è il massimo valore di velocità che un oggetto fisico (corpo, corpuscolo od onda) può possedere, ed è costante anche al variare dei sistemi di riferimento.

E' però possibile che due corpi si allontanino fra loro a velocità maggiori di c, ma non per loro virtù, semplicemente perchè lo spazio si espande. Questo comunque è quello che ho recepito.

Non è che la cosa mi convinca appieno, perché fatico a capire che differenza ci sia fra una galassia che si muova per moto proprio, magari come effetto tardivo del big bang, ed una che ha un moto apparente dovuto all'espansione dello spazio. Soprattutto non capisco come si faccia a valutare tale differenza.

Ad esempio, prendendo per buona una frase di WP, si ha:
Due astronavi, ognuna viaggiante al 90% della velocità della luce relativamente a un osservatore posto tra di esse, non si percepiscono l’un l’altra come in avvicinamento al 180% della velocità della luce. La velocità apparente è comunque inferiore al 100% della velocità della luce.

Ora, se ognuna di queste due astronavi viaggia all'interno di ognuna di quelle due galassie con una velocità pari a quella del loro allontanamento per l'effetto di espansione, immagino che verranno viste come ferme rispetto alle relative galassie. Un po' come due zanzare che volino su due punti del palloncino con una velocità uguale all'allontanamento di quei due punti.
Ma allora, come mai le due astronavi continuerò a vederle, mentre le galassie spariranno? In altre parole, come mai per le galassie, che rispetto ad un osservatore fisso sono in movimento, vale la familiare proprietà additiva delle velocità, mentre per le astronavi devo usare l'additività relativistica, pur muovendosi allo stesso modo?

DarknessLight
15-05-2015, 23:25
Marcos64 l universo (lo spazio tempo) si espande a velocità superiori di quelle della luce!!! Gli oggetti all interno dell universo invece no, viaggiano al massimo alla velocità c (o si approssimano ad essa), ma l espansione dello spazio tempo fa si che ci sia un APPARENTE spostamento reciproco a velocità superiori a quelle della luce
Come detto precedentemente, oltre la sfera di hubble (di raggio 46 miliardi di anni luce se non erro) gli oggetti si muovono RISPETTO A NOI a velocità molto superiori di quelle della luce, ma il motivo di questa espansione a velocità superiori a quelle della luce è una caratteristica fisica del nostro universo ;)

DarknessLight
15-05-2015, 23:32
Inoltre la sfera di hubble ha raggio di 46 miliardi di anni luce perchè se ci pensate gli oggetti che noi vediamo come i più vecchi, noi li vediamo come erano 13,7 miliardi di anni fa, ma oggi essi si sono allontanati da noi in seguito all espansione dello spazio, quindi ora si trovano a 46 miliardi di anni luce da noi
Un giorno non li vedremo più perché l espansione li trascinera via a velocità superiori a quelle della luce e non potremo più ricevere da loro informazione elettromeagnetica ;)

Marcos64
15-05-2015, 23:36
Allora non ero lontano dal vero, nessuna particella mai superera' c, solo il reciproco allontanamento, produce il fatto che i due oggetti, non potranno vedersi mai. O sbaglio?

DarknessLight
15-05-2015, 23:44
Giusto!!!
All interno dell universo NIENTE si muove più veloce della luce!!! È l espansione dello spazio tempo che avviene a velocità superiori a quelle della luce (non mi chiedere il perché :razz:) e di conseguenza gli oggetti che si trovano reciprocamente oltre la propria sfera di hubble si muovono di moto reciproco superiore alla velocità della luce. Ma ripeto, non è l oggetto a superare c, bensì è lo spazio tempo a espandersi molto molto velocemente... e questa espansione sta anche accelerando!!!!
Magari poi Enrico e Red potranno darvi una panoramica migliore della questione... questo è solo il succo essenziale ;)

Enrico Corsaro
16-05-2015, 00:02
Ringrazio Marcos64 per il riferimento!

Stiamo già discutendo la cosa abbastanza in dettaglio in questo (http://www.astronomia.com/forum/showthread.php?10726-Il-modello-cosmologico-standard-%CE%9BCDM-Parte-II-quanta-materia-ed-energia-oscura) post nel forum Articoli, se siete interessati al problema vi consiglio di dare un occhio.

Sostanzialmente non bisogna fare confusione tra velocità di spostamento di un corpo rispetto al suo sistema di riferimento, e velocità di spostamento dello stesso sistema di riferimento rispetto ad un altro sistema di riferimento.
Nel primo caso effettivamente il corpo non può superare in alcun modo la velocità della luce nel vuoto (che specificiamo, nel vuoto, è appunto una costante). Nel secondo caso invece parliamo di un sistema di riferimento che non è un oggetto fisico vero e proprio, è solo un riferimento spaziale. In questo caso non vale più il limite della velocità della luce. Questo implica chel'oggetto potrebbe essere anche perfettamente fermo nel suo sistema di riferimento ma che il sistema di riferimento in cui si trova si può spostare alla velocità che vuole. I segnali elettromagnetici si propagheranno sempre e comunque a velocità c.
Quando è lo spazio-tempo ad espandersi, non stiamo parlando di un oggetto sottoposto alle leggi fisiche che conosciamo, ma dello stesso spazio in cui viviamo. Il sistema di riferimento solidale con noi costituirà la regione di spazio vicina a noi, mentre un'altra regione di spazio prossima alla sorgente che osserviamo sarà un'altro sistema di riferimento. Può accadere che la sorgente si muova dentro quel suo sistema di riferimento, o anche no. E può accadere, come vediamo, che quel sistema di riferimento si muove rispetto al nostro! In quest'ultimo caso, la velocità di moto non è sottoposta ad alcun vincolo fisico.

Arzak
16-05-2015, 00:07
Nota tecnica: per chi è interessato, ho modificato il post precedente grazie alla nascita di nuovi dubbi.

Marcos64
16-05-2015, 00:26
Grazie, credo di aver capito, certo ci mettero' tempo a metabolizzare questo nuovo concetto.

Enrico Corsaro
16-05-2015, 01:25
Aspetta Arzak, il modo in cui hai posto il problema non è molto chiaro. Ciò che confondi è la velocità con cui viaggia l'astronave con quella con cui invece si espande l'Universo. Sono due cose molto diverse e ti prego di leggere il mio post precedente se non lo hai già fatto.



Ad esempio, prendendo per buona una frase di WP, si ha:
Due astronavi, ognuna viaggiante al 90% della velocità della luce relativamente a un osservatore posto tra di esse, non si percepiscono l’un l’altra come in avvicinamento al 180% della velocità della luce. La velocità
apparente è comunque inferiore al 100% della velocità della luce.
Qui entrano in gioco le correzioni relativistiche, e devi applicare la somma di velocità della relatività ristretta, perchè viaggiando al 90% della velocità della luce già risenti degli effetti relativistici. Quello che succede è che le due componenti si sommano, dando luogo a 1.8c (180% della velocità della luce), ma vengono riscalate per un fattore di correzione che nel qual caso è dato da 1 + (0.9c * 0.9c / c2). Il risultato è che la velocità finale con cui si vedono le due astronavi è 0.994475 c, quindi non superiore a quella della luce.



Ora, se ognuna di queste due astronavi viaggia all'interno di ognuna di quelle due galassie con una velocità pari a quella del loro allontanamento per l'effetto di espansione, immagino che verranno viste come ferme rispetto alle relative galassie. Un po' come due zanzare che volino su due punti del palloncino con una velocità uguale all'allontanamento di quei due punti.
Ma allora, come mai le due astronavi continuerò a vederle, mentre le galassie spariranno? In altre parole, come mai per le galassie, che rispetto ad un osservatore fisso sono in movimento, vale la familiare proprietà additiva delle velocità, mentre per le astronavi devo usare l'additività relativistica, pur muovendosi allo stesso modo?

Il motivo è semplice in un certo senso. Mentre nel caso iniziale che poni le due astronavi continuano a vedersi perchè effettivamente non viaggiano, nè possono, più veloce della luce, nel secondo caso, la velocità di espansione che le allontana è realmente più veloce della luce, e questo fa si che un segnale elettromagnetico emesso da una delle due non potrà mai raggiungere l'altra. Quindi le astronavi non possono di per sè viaggiare più veloce della luce, ma lo spazio su cui giacciono può spostarsi più veloce della luce e dunque trascinarsele dietro.

Morimondo
16-05-2015, 01:57
Sto digerendo e smaltendo con molta fatica questa espansione dello spazio a velocita superiori a quella della luce, sul web ho trovato il termine "espansione metrica dello spazio" di Friedmann, Friedman era russo e scrisse in tedesco il suo lavoro che rimase così inosservato. Mori nel 1988.

Un articolo su le scienze diceva che si stava valutando se la gravità esercitata dalla massa di tutte le galassie dell'universo fosse o no sufficiente a trattenenere fino a frenare e invertire l'espansione fino al ritorno alla singolarità del big bang. Sono quasi certo di aver letto questo articolo negli anni 90 quindi dopo la morte di friedman, in sostanza da quando e chi ha riscoperto e divulgato il suo lavoro?

Enrico Corsaro
16-05-2015, 02:25
Il lavoro di Friedmann è noto dagli anni '20. Egli trovò la prima soluzione alle equazioni di campo di Einstein nel 1922, quindi è conosciuto già dagli albori della relatività generale. Non fu inizialmente acclamato perchè mancavano ancora le evidenze osservative di un universo dinamico e perchè come dici tu il lavoro fu scritto in russo, quindi sostanzialmente molto poco letto in altre parti del mondo dove i luminari della scienza operavano.
L'espansione dell'Universo fu invece scoperta osservativamente da Hubble, quando nel 1929 pubblicò per la prima volta che galassie diverse dalla nostra si allontanavano non per una velocità propria ma per effetto della espansione dello stesso Universo.

Arzak
16-05-2015, 13:08
Mentre nel caso iniziale che poni le due astronavi continuano a vedersi perchè effettivamente non viaggiano, nè possono, più veloce della luce, nel secondo caso, la velocità di espansione che le allontana è realmente più veloce della luce, e questo fa si che un segnale elettromagnetico emesso da una delle due non potrà mai raggiungere l'altra. Quindi le astronavi non possono di per sè viaggiare più veloce della luce, ma lo spazio su cui giacciono può spostarsi più veloce della luce e dunque trascinarsele dietro.
Immagino che tutti comprendano che concetti così al di fuori della nostra esperienza personale siano difficili da digerire. Spero dunque che la riproposta dei miei dubbi, che a quanto pare non sono così rari in questo campo, non venga presa per un'irragionevole ostinazione. Ho letto con attenzione gli interventi degli amici, intuisco in qualche modo che le loro risposte sono coerenti e ragionevoli, ma i dubbi permangono, e cerco di chiarirli con un disegnino:

http://i19.servimg.com/u/f19/17/25/68/48/espans10.jpg

L'astronave A viaggia al di sopra della galassia A con una certa velocità. Se questa velocità è uguale a quella, sia pure apparente, con cui la galassia si muove per effetto della dilatazione dell'universo, la galassia dovrebbe vedere l'astronave ferma al di sopra di essa. Questa era la mia prima ipotesi, ma Enrico mi fa giustamente notare che anche l'astronave subisce l'effetto di dilatazione. A questo punto mi chiedo: come si muove in realtà l'astronave rispetto alla galassia?

Se le due astronavi sono (appaiono) immobili sopra le rispettive galassie, mi riusciva difficile capire come mai ad un certo punto le galassie scomparivano l'una rispetto all'altra per via del superamento di c, mentre le astronavi, per la formula relativistica ricordata sopra rimanevano visibili.
Ora mi pare invece di capire che anche le astronavi, muovendosi sia coi propri mezzi che per via dell'espansione, dovrebbero anche loro scomparire alla vista. Ho capito bene?

In ogni caso, faccio ancora fatica ad accettare (se ciò che ho capito è vero) che per le galassie le rispettive velocità, pur prossime a c, vadano sommate algebricamente mentre per le astronavi valga l'additività relativistica. Ma sono sicuro che qualcosa ancora mi sfugge.



PS: Ho omesso la questione dell'aumento di massa e dell'accorciamento degli oggetti che viaggiano a velocità prossima a c. Spero che la cosa sia ininfluente ai fini di quanto diciamo, ma se non lo fosse mi chiedo di nuovo come mai, sempre se ho capito bene, il fenomeno vale solo per oggetti dotati di moto proprio e non per quelli la cui velocità è solo dovuta all'espansione dello spazio. Chi glielo dice ad un osservatore esterno che un oggetto ha una velocità propria mentre l'altro no, dal momento che li vede muoversi entrambi?

Enrico Corsaro
16-05-2015, 19:06
Arzak il disegnino è molto simpatico, disegnerei però l'effetto dell'espansione con delle frecce al centro a forma di croce, per indicare che l'espansione avviene verso tutte le direzioni.

Adesso che hai riscritto penso mi sia chiaro cosa intendi dire. Vediamo di spiegarlo ma cerchiamo di semplificarci la vita e lasciamo perdere il moto a velocità prossime a c delle astronavi per il momento..che non è collegato in alcun modo con l'espansione.



L'astronave A viaggia al di sopra della galassia A con una certa velocità. Se questa velocità è uguale a quella, sia pure apparente, con cui la galassia si muove per effetto della dilatazione dell'universo, la galassia dovrebbe vedere l'astronave ferma al di sopra di essa. Questa era la mia prima ipotesi, ma Enrico mi fa giustamente notare che anche l'astronave subisce l'effetto di dilatazione. A questo punto mi chiedo: come si muove in realtà l'astronave rispetto alla galassia?

Immaginiamo che l'astronave sia ferma, cioè abbia i motori spenti. Non devi immaginare che la galassia o qualsiasi altro oggetto si muova per effetto dell'espansione. L'espansione non fa muovere niente al suo interno con moto proprio, è un effetto diciamo apparente quello del moto visibile tra due oggetti a distanza. Dunque se l'astronave è dentro la galassia o in prossimità ed è ferma, essa continuerà a rimanere ferma rispetto alla galassia. Se l'astronave è comunque ferma, ma è abbastanza lontana dalla galassia, allora vedrà la galassia allontanarsi perchè lo spazio interposto diventa grande abbastanza da rendere visibile l'espansione. Ma nè la galassia nè l'astronave si muovono realmente. E' un pò come se tu ed un'altra persona foste in piedi dentro una stanza e vi guardate in faccia. Ad un certo punto il pavimento sotto di voi comincia ad estendersi, e voi vi allontanate. Sarete comnque fermi dal vostro punto ma vedrete l'altro allontanarsi da voi.



Se le due astronavi sono (appaiono) immobili sopra le rispettive galassie, mi riusciva difficile capire come mai ad un certo punto le galassie scomparivano l'una rispetto all'altra per via del superamento di c, mentre le astronavi, per la formula relativistica ricordata sopra rimanevano visibili.
Ora mi pare invece di capire che anche le astronavi, muovendosi sia coi propri mezzi che per via dell'espansione, dovrebbero anche loro scomparire alla vista. Ho capito bene?

Stavamo parlando di due casi distinti infatti. Se due astronavi sono effettivamente in moto a velocità prossime della luce perchè i loro motori lo permettono, non riusciranno mai a superare c e continueranno a vedersi sempre.
Tuttavia, se le astronavi sono così distanti da far si che l'espansione che avviene nello spazio tra di loro raggiunga velocità superluminali, che le astronavi siano ferme o in moto con i loro motori non importa, ad un certo punto non potranno più vedersi. Stesso discorso vale per le galassie.



In ogni caso, faccio ancora fatica ad accettare (se ciò che ho capito è vero) che per le galassie le rispettive velocità, pur prossime a c, vadano sommate algebricamente mentre per le astronavi valga l'additività relativistica. Ma sono sicuro che qualcosa ancora mi sfugge.

Infatti non è affatto così, sia per le galassie che per le astronavi valgono gli stessi identici principi, solo che è praticamente impossibile nella realtà che una galassia si muova di suo a velocità prossime a c perchè non ci sono condizioni energetiche nella formazione della galassia perchè ciò si verifichi. Una astronave al limite OK, se parliamo un pò di fantascienza diciamo. Ma questo non ha nulla a che vedere con l'espansione dello spazio, che è una cosa completamente a sè stante.
Dunque quando parliamo di moto vero e proprio, quello per cui serve l'energia cinetica per intenderci, allora non si può superare c, e i vettori velocità si sommano secondo la relatività ristretta.
Quando invece parliamo di espansione dello spazio, ciò che entra in gioco è la relatività generale e siccome lo spazio non è nè un segnale, nè un oggetto dotato di massa, non ha alcun vincolo con la velocità della luce.

Ricordo infatti che il postulato sulla velocità della luce afferma che qualsiasi segnale elettromagnetico (cioè i fotoni) viaggiano a quella velocità e che nessun corpo dotato di massa può raggiungerla. Il resto, non rientra in questo postulato ed ecco perchè lo spazio, che non è nè un segnale nè un oggetto dotato di massa, ha la possibilità di espandersi a velocità arbitrariamente elevate, non c'è alcun limite in questo caso.




PS: Ho omesso la questione dell'aumento di massa e dell'accorciamento degli oggetti che viaggiano a velocità prossima a c. Spero che la cosa sia ininfluente ai fini di quanto diciamo, ma se non lo fosse mi chiedo di nuovo come mai, sempre se ho capito bene, il fenomeno vale solo per oggetti dotati di moto proprio e non per quelli la cui velocità è solo dovuta all'espansione dello spazio. Chi glielo dice ad un osservatore esterno che un oggetto ha una velocità propria mentre l'altro no, dal momento che li vede muoversi entrambi?
Stiamo parlando in questo contesto di velocità, cioè di lunghezze percorse per unità di tempo, dunque l'accorciamento, che avviene solo sul corpo stesso in moto e non sullo spazio (non c'è motivo che lo spazio ne risenta se è solo il corpo al suo interno a viaggiare a velocità così alte), non influisce sul discorso, nè tantomeno la massa in questo contesto. Evitiamo di complicarci la vita se il concetto di base non è ancora chiaro ;) e andiamo per gradi.

Il modo per capire se un oggetto ha velocità propria oppure è soggetto all'espansione è tramite studi sistematici. Puoi risalire alle proprietà cinematiche di varie regioni dell'Universo tramite studi astrometrici, spettroscopici e dinamici, e da li puoi vedere se c'è qualcosa di residuo che appare. Inoltre, l'espansione di per sè è un effetto che appare uguale in tutte le direzioni, quindi è relativamente semplice rendersene conto se si riesce a misurare la distanza degli oggetti. Per misurare la distanza si usano ad esempio cefeidi e supernovae. Si costruisce una mappa di velocità in tutte le direzioni e si vede come effettivamente c'è un denominatore comunque, cioè una velocità di allontanamento che cresce con la distanza in tutte le direzioni. Ti chiederai allora perchè non pensare che il problema nasce dal nostro punto di osservazione. Perchè gli oggetti vicini, le galassie vicine, le vediamo anche avvicinarsi a noi e abbiamo una idea abbastanza dettagliata di ciò che sono i moti nei dintorni della nostra galassia. Ciò, insieme alla distribuzione delle galassie più lontane, ci porta a ritenere che non siamo speciali rispetto ad altri punti di osservazione.

DarknessLight
16-05-2015, 19:37
Arzak Un piccolo inciso che non risponde certo alla complicata domanda che poni ma che può forse aiutarti a capire meglio la questione: quando si parla di espansione dello spazio-tempo (che come abbiamo detto oltre la sfera di hubble avviene a velocità superiori a quelle della luce) è un effetto che si rende evidente per distanze cosmologiche o comunque nell ordine di qualche decina di milione di anni luce o giù di lì.
Per le piccole distanze (come possono essere quelle galattiche o degli ammassi di galassie) l espansione quasi non si sente e sicuramente non se ne percepisce nemmeno l accelerazione.
Con questo cosa voglio dire?
Voglio dire che i sistemi stellari, le galassie e gli ammassi di galassie sono legati dalla mutua attrazione gravitazionale, perciò sono considerabili come oggetti a se è le stelle al loro interno non si allontanano le une dalle altre poiché per piccole scale prevale la gravità che tiene uniti i corpi.

davide1334
16-05-2015, 19:54
vorrei chiedere anch'io una cosa,sperando sia una domanda sensata: l'espansione avviene in tutte le direzioni,ma all'interno delle strutture legate gravitazionalmente (galassie,gruppi,ammassi)non viene percepita giusto? cioè la gravità la contrasta e gli avvicinamenti o allontanamenti dipendono solo dal moto prorio degli oggetti.a quali ordini di distanze o di "raggruppamenti" essa prende il sopravvento in modo da essere percepita? vi è un limite oppure no?

ps : ah,leggo solo ora l'intervento di darkness....telepatia?:biggrin:

Red Hanuman
16-05-2015, 19:57
Magari, un esempio in più serve a chiarire le idee a Arzak.
Tieni dunque conto che l'espansione cosmologica è molto piccola, e comincia ad essere rilevante per distanze di milioni di a.l. .
Quindi, fintanto che il tasso di espansione rimane al di sotto di certi limiti, la gravità e le altre forze fondamentali tengono insieme la materia e le galassie stesse con i loro ammassi (di un eventuale big rip (http://it.wikipedia.org/wiki/Big_Rip) parleremo un'altra volta...;))

Immagina, allora, che lo spazio sia la classica tovaglia quadrettata. In questo caso, i quadretti sono grandi come un'intera galassia, (perchè tutta la galassia e quello che essa contiene si mantengono unite per via delle forze fondamentali). Anzi, meglio ancora, un quadrato contiene tutto quello che rimane legato gravitazionalmente.
Fintanto che le astronavi continuano a muoversi nello stesso quadretto, vale la sola RG, per cui la composizione delle velocità tra le due si mantiene al di sotto di C.
Se però le due astronavi sono su due galassie lontane, tra i due quadretti in cui sono contenute se ne può formare un altro per via dell'espansione dello spazio. Dopo un po' se ne forma un'altro, e un'altro ancora, e così via....
Se io non sapessi che lo spazio si sta formando tra le due galassie, cosa penserei? Che le due astronavi viaggiano ad una velocità superiore a quella della luce, e che quindi la RG non vale più. Ma è vero?:sneaky:
NO. Debbo tenere conto che creazione di spazio e movimento NELLO spazio sono due cose completamente diverse.
Se voglio conoscere il tasso di movimento NELLO spazio, allora alla velocità apparente debbo sottrarre il tasso di formazione di spazio tra le due astronavi. Se lo faccio, i conti tornano SEMPRE, e la composizione delle velocità mi da SEMPRE valori minori od al limite uguali a C.;)

Enrico Corsaro
16-05-2015, 19:58
vorrei chiedere anch'io una cosa,sperando sia una domanda sensata: l'espansione avviene in tutte le direzioni,ma all'interno delle strutture legate gravitazionalmente (galassie,gruppi,ammassi)non viene percepita giusto? cioè la gravità la contrasta e gli avvicinamenti o allontanamenti dipendono solo dal moto prorio degli oggetti.a quali ordini di distanze o di "raggruppamenti" essa prende il sopravvento in modo da essere percepita? vi è un limite oppure no?

Esatto non viene percepita per scale relativamente piccole. Il limite ben visibile è grossomodo posto oltre l'ammasso galattico della Vergine, intorno a 30 milioni di anni luce di distanza. Comunque misure più accurate e che tolgono l'effetto degli altri moti locali, riescono a misurare l'espansione già nell'ordine del milione di anni luce di distanza.

Red Hanuman
16-05-2015, 19:59
vorrei chiedere anch'io una cosa,sperando sia una domanda sensata: l'espansione avviene in tutte le direzioni,ma all'interno delle strutture legate gravitazionalmente (galassie,gruppi,ammassi)non viene percepita giusto? cioè la gravità la contrasta e gli avvicinamenti o allontanamenti dipendono solo dal moto prorio degli oggetti.a quali ordini di distanze o di "raggruppamenti" essa prende il sopravvento in modo da essere percepita? vi è un limite oppure no?
Mi sa che in parte ti ho già risposto. Il limite a cui ti riferisci dipende dal tasso di espansione. Se effettivamente sta aumentando, allora pian piano tutto si disgregarà, fino al Big Rip.... ;)

Arzak
16-05-2015, 20:36
Se voglio conoscere il tasso di movimento NELLO spazio, allora alla velocità apparente debbo sottrarre il tasso di formazione di spazio tra le due astronavi. Se lo faccio, i conti tornano SEMPRE, e la composizione delle velocità mi da SEMPRE valori minori od al limite uguali a C.;)
Era appunto ciò che intendevo chiedere già da prima, ma non osavo fare per rispetto della memoria di Einstein. Ciò che dici, e cioé che i conti vanno rifatti, però mi conforta e mi spinge all'azzardo: ma allora, se è vero che comunque le astronavi di cui parlavo e che cita WP ad un certo punto non si vedono più fra di loro nonostante la formula che dice che viaggiano ad una velocità relativa inferiore a C, non sarebbe il caso di correggere l'espressione con un fattore che tenga conto dell'espansione dello spazio ottenendo così una formulazione più generale del problema?

FT: Ne approfitto per ringraziare di cuore collettivamente tutti gli amici che sono intervenuti con una pazienza che loro invidio. Faccio inoltre i miei complimenti ad un forum così ricco e interessante, e pieno di persone così colte e disponibili.

Enrico Corsaro
16-05-2015, 20:53
Era appunto ciò che intendevo chiedere già da prima, ma non osavo fare per rispetto della memoria di Einstein. Ciò che dici, e cioé che i conti vanno rifatti, però mi conforta e mi spinge all'azzardo: ma allora, se è vero che comunque le astronavi di cui parlavo e che cita WP ad un certo punto non si vedono più fra di loro nonostante la formula che dice che viaggiano ad una velocità relativa inferiore a C, non sarebbe il caso di correggere l'espressione con un fattore che tenga conto dell'espansione dello spazio ottenendo così una formulazione più generale del problema?


L'unica correzione da fare, se si è interessati ad ottenere il moto proprio, è semplicemente togliere l'effetto dell'espansione con una sottrazione, nè più nè meno ;), quindi non occorre riscrivere la teoria per questo. Sappiamo bene di che effetto si tratta e sappiamo come tenerlo in considerazione. In realtà comunque le velocità nel contesto cosmologico non sono un tipo di dato che viene utilizzato per vincolare i modelli e all'atto pratico non abbiamo mai a che fare con velocità superluminali (altrimenti non vedremmo le sorgenti). I dati utilizzati sono il redshift e le distanze degli oggetti, quindi sostanzialmente non occorre preoccuparsi ulteriormente da questo punto di vista.

Red Hanuman
16-05-2015, 23:48
Adesso, caro Enrico Corsaro, permettimi una digressione a livello filosofico - fantascientifico: perchè l'espansione cosmologica deve essere SOLO dello spazio? In fondo spazio e tempo, come diceva il vecchio Abert, sono legati a doppio filo tra di loro.
Quindi, se fosse creazione di spazio E tempo? Ovvero, se lo scorrere della dimensione tempo fosse legato alla creazione della dimensione spazio?:thinking: Chissà.... ;):razz:

Arzak
17-05-2015, 00:08
(La butto lì, tanto per dire...) http://r15.imgfast.net/users/1517/32/07/50/smiles/2774597380.gif
Una "creazione" di spazio significa che il volume dell'universo aumenta. Una "creazione" di tempo credo vada letta come un rallentamento del tempo. Dal momento che lo spazio si espande a velocità "luminose", le due cose starebbero anche assieme, secondo la relatività.
Se solo lo spazio fosse un oggetto, il ché non è, tanto è vero che può viaggiare anche a velocità > c.
Però...

Enrico Corsaro
17-05-2015, 06:10
Adesso, caro @Enrico Corsaro (http://www.astronomia.com/forum/member.php?u=2649), permettimi una digressione a livello filosofico - fantascientifico: perchè l'espansione cosmologica deve essere SOLO dello spazio? In fondo spazio e tempo, come diceva il vecchio Abert, sono legati a doppio filo tra di loro.
Quindi, se fosse creazione di spazio E tempo? Ovvero, se lo scorrere della dimensione tempo fosse legato alla creazione della dimensione spazio?:thinking: Chissà.... ;):razz:

Un ottimo spunto di riflessione @Red Hanuman (http://www.astronomia.com/forum/member.php?u=9), hai fatto bene a tirarlo in ballo e provo a risponderti senza dover entrare nella fantascienza :razz:. La risposta infatti è già racchiusa in ciò che conosciamo.

Nel modello cosmologico l'effetto dell'espansione si riflette unicamente sullo spazio (cioè sulle coordinate spaziali) ma è funzione soltanto del tempo, cioè è visibile solo grazie al fatto che il tempo trascorre. Cerco di spiegarmi meglio. Considera l'Universo per un momento come se fosse statico, cioè senza una evoluzione. Rappresentiamo questo universo con sole coordinate spaziali, senza dunque bisogno del tempo.
Cosa c'è in più dunque nell'Universo che conosciamo? Esattamente il tempo. E' il tempo che ci permette di rappresentare l'evoluzione dello spazio, ed è qui che entra in gioco il legame tra le tre coordinate spaziali e quella temporale in cosmologia, cioè il fatto che tempo e spazio siano intimamente legati. Lo spazio non cambia e non evolve nè può evolvere se il tempo non trascorre.

Diverso il caso in cui invece opera il relativismo, cioè in cui il tempo viene deformato e rallentato per effetto di un intenso campo gravitazionale (o in alternativa di una velocità di moto prossima a c). In questo caso bisogna prestare attenzione a quale punto di osservazione si considera. Esiste sempre un punto di vista esterno, assoluto, e quello coincide con il tempo cosmologico, che chiamiamo tempo cosmico standard e che non cambia. Questo tempo non è mai soggetto a variazioni causate da ciò che c'è dentro l'Universo, perchè è lo stesso tempo dell'Universo. Invece il tempo di un osservatore soggetto a forti campi gravitazionali si dilata e questo perchè questo tempo non coincide più con un punto di vista assoluto, cioè con il tempo cosmico, ma diventa relativo, cioè apparente per quell'osservatore.

Adesso però entro in un campo pericoloso, ciò che non è provato scientificamente nè è provabile scientificamente ma che rimane comunque un discorso logico.
La domanda che sorge a questo punto è perchè il tempo cosmico non cambia nè può cambiare? Perchè non esiste alcun modo di verificarlo. Anche se dovesse cambiare, non ce ne accorgeremmo. E perchè non possiamo accorgecene? Perchè non abbiamo un riferimento ESTERNO all'Universo, cioè un riferimento assoluto di tempo rispetto al tempo dell'Universo. Se reputiamo che l'Universo sia tutto ciò che esiste, questo riferimento esterno non esiste neanche e di conseguenza il tempo dell'Universo è il riferimento assoluto, e non c'è motivo per cui debba cambiare.

Per analogia immaginate (sforziamoci un pò, è solo un esempio per capire il discorso, non prendetelo alla lettera!!) che il vostro Universo sia costituito da uno spazio in cui vivete, diciamo una stanza a forma di cubo, e che sia posta in prossimità di un enorme buco nero. Allora in quel caso il vostro tempo non è, nè può essere assoluto perchè avrete il riferimento del tempo che scorre al di fuori del vostro Universo, cioè al di fuori della vostra stanza lontano dal buco nero. In questo caso vi potete rendere conto che il tempo che vivete sta di fatto rallentando per effetto del campo gravitazionale.

Spero di essermi fatto capire... :awesome:

DarknessLight
17-05-2015, 14:10
Il tempo assoluto è il tempo cosmico standard.
Lo spazio assoluto qual è?
Certo, mi risponderete che sono le altre tre coordinate assolute cosmologiche, ma che cosa mi rappresentano queste coordinate? Se l universo ha un tempo ben definito (13,7 miliardi) allora ha anche un volume ben definito?

Enrico Corsaro
17-05-2015, 16:45
Il tempo assoluto è il tempo cosmico standard.
Lo spazio assoluto qual è?
Certo, mi risponderete che sono le altre tre coordinate assolute cosmologiche, ma che cosa mi rappresentano queste coordinate? Se l universo ha un tempo ben definito (13,7 miliardi) allora ha anche un volume ben definito?

Le tre coordinate spaziali non vengono chiamate assolute perchè sono comoventi, cioè non dipendono dal tempo. Per avere le coordinate assolute devi includere l'effetto temporale, dunque inglobare il fattore di scala cosmico (come un pò spiegato nell'articolo che ho pubblicato). Queste coordinate ti descrivono la posizione di un qualunque oggetto all'interno dell'Universo, fissata l'origine del sistema di riferimento su di noi per comodità.
Come hai giustamente detto, un tempo ben definito corrisponde ad un volume ben definito. Le coordinate sono altamente legate fra loro.

Enrico Corsaro
19-05-2015, 04:10
DarknessLight forse questa ti era sfuggita, ti ho taggato così dovresti vederla ;).

DarknessLight
19-05-2015, 09:36
Sì grazie Enrico Corsaro in effetti non l avevo vista ;)

Comunque pensare ad un volume ben definito di universo non è un po strano? Voglio dire, ad oggi conosciamo il volume esatto dell universo? Sapendo il tempo assoluto suppongo che si dovrebbe conoscere anche il volume... a meno che l energia oscura è l inflazione non abbiano fatto scherzi particolari....

Enrico Corsaro
19-05-2015, 09:45
Comunque pensare ad un volume ben definito di universo non è un po strano? Voglio dire, ad oggi conosciamo il volume esatto dell universo? Sapendo il tempo assoluto suppongo che si dovrebbe conoscere anche il volume... a meno che l energia oscura è l inflazione non abbiano fatto scherzi particolari....

Se puoi definire un tempo assoluto puoi definire anche uno spazio assoluto, perchè no, alla fine è pur sempre un set di dimensioni che possiamo misurare...il punto è che gli assoluti sono comunque definiti sempre da un modello, ma non abbiamo una misura vera, nè mai la avremo, del tempo di vita dell'Universo e del suo volume. La nostra comprensione della realtà rimane pur sempre una approssimazione, se pur resa quanto più realistica possibile.

DarknessLight
19-05-2015, 12:39
Cosa significa che non avremo mai una misura VERA dell età e delle dimensioni dell universo?
Anche delle stelle facciamo misure indirette. Anche dei buchi neri. Anche la teoria dell evoluzione è per certi versi spiegata indirettamente. Non è che andiamo sulle stelle o sui buchi neri a misurarli o andiamo indietro nel tempo a vedere le stromatoliti. Eppure queste teorie vanno bene lo stesso. Anche per l universo dovrebbe essere così o sbaglio?

Comunque visto che stiamo parlando di volume, ad oggi a quanto è stimato il volume dell universo(anche se si tratta di una stima approssimativa)?

davide1334
19-05-2015, 14:30
forse la domanda non ha senso in partenza,comunque vabbè... il nostro universo osservabile,la sfera di hubble è appunto una sfera...l'universo "tutto" è plausibile pensare che sia anch'esso una sfera(ovviamente molto più grande) o non necessariamente?è improprio parlare di "forma"?

Enrico Corsaro
19-05-2015, 18:59
forse la domanda non ha senso in partenza,comunque vabbè... il nostro universo osservabile,la sfera di hubble è appunto una sfera...l'universo "tutto" è plausibile pensare che sia anch'esso una sfera(ovviamente molto più grande) o non necessariamente?è improprio parlare di "forma"?
davide1334 ti invito a leggere questo (http://www.astronomia.com/2015/04/22/il-modello-cosmologico-standard-%CE%BBcdm-parte-i-cose-e-come-si-ricava/) articolo, ne abbiamo discusso ampiamente in merito ;).

Enrico Corsaro
19-05-2015, 19:19
Cosa significa che non avremo mai una misura VERA dell età e delle dimensioni dell universo?
Anche delle stelle facciamo misure indirette. Anche dei buchi neri. Anche la teoria dell evoluzione è per certi versi spiegata indirettamente. Non è che andiamo sulle stelle o sui buchi neri a misurarli o andiamo indietro nel tempo a vedere le stromatoliti. Eppure queste teorie vanno bene lo stesso. Anche per l universo dovrebbe essere così o sbaglio?

Comunque visto che stiamo parlando di volume, ad oggi a quanto è stimato il volume dell universo(anche se si tratta di una stima approssimativa)?

Una cosa è la realtà, un'altra cosa è l'interpretazione che ne diamo. Le due cose non coincidono perfettamente perchè nel secondo caso è il nostro modo di spiegare la realtà, che è pur sempre una approssimazione.
Tutto ciò che misuriamo è soggetto ad incertezza, sia di natura teorica che strumentale. Le teorie vanno bene relativamente al livello di precisione che intendi raggiungere, ma non potranno mai essere esatte (matematicamente parlando). Questa è la fisica...

Comunque ovviamente il volume si può calcolare se fai delle considerazioni sul raggio, basta anche fare un pò di ricerca in giro per trovare delle stime. Se consideri un raggio di universo osservabile (cioè tutto l'Universo a partire dal Big Bang in sostanza), ottieni un valore di circa 47 miliardi di anni luce. Questo ti da un volume di 4 x 1080 metri cubi. Direi comunque che non ce ne facciamo molto di questi valori alla fine poichè sono solo estrapolazioni :). Hanno l'utilità unica di darci una idea di quanto grande possa essere, ma non aggiungono alcuna informazione sensibile alla fisica del cosmo.

DarknessLight
19-05-2015, 19:50
Una cosa è la realtà, un'altra cosa è l'interpretazione che ne diamo. Le due cose non coincidono perfettamente perchè nel secondo caso è il nostro modo di spiegare la realtà, che è pur sempre una approssimazione.
Tutto ciò che misuriamo è soggetto ad incertezza, sia di natura teorica che strumentale. Le teorie vanno bene relativamente al livello di precisione che intendi raggiungere, ma non potranno mai essere esatte (matematicamente parlando). Questa è la fisica...

In pratica mi stai dicendo che le stime che facciamo sono delle approssimazioni più o meno veritiere della realtà. E quindi anche l età dell universo potrebbe essere sbagliata. È questo che vuoi dire?


Comunque ovviamente il volume si può calcolare se fai delle considerazioni sul raggio, basta anche fare un pò di ricerca in giro per trovare delle stime. Se consideri un raggio di universo osservabile (cioè tutto l'Universo a partire dal Big Bang in sostanza), ottieni un valore di circa 47 miliardi di anni luce. Questo ti da un volume di 4 x 1080 metri cubi. Direi comunque che non ce ne facciamo molto di questi valori alla fine poichè sono solo estrapolazioni :). Hanno l'utilità unica di darci una idea di quanto grande possa essere, ma non aggiungono alcuna informazione sensibile alla fisica del cosmo.

E questo ok, il raggio dell universo osservabile lo conoscevo, ma esistono anche stime del volume dell INTERO universo?

Enrico Corsaro
19-05-2015, 20:03
In pratica mi stai dicendo che le stime che facciamo sono delle approssimazioni più o meno veritiere della realtà. E quindi anche l età dell universo potrebbe essere sbagliata. È questo che vuoi dire?
Non diciamo sbagliata perchè vorrebbe dire che tutto il lavoro fatto sia privo di senso...e non lo è, però l'età può essere imprecisa, potrebbe essere soggetta a variazioni, effetti sistematici che non abbiamo contemplato. E comunque l'età dipende dal modello usato, e se cambi il modello l'età cambia!
Non abbiamo alcun modo di sapere realmente quanto l'Universo sia vecchio, se non estrapolandolo dai modelli.



E questo ok, il raggio dell universo osservabile lo conoscevo, ma esistono anche stime del volume dell INTERO universo?
Te l'ho già scritto il numero :sneaky:, ti invito a rileggere meglio il mio post.

DarknessLight
19-05-2015, 20:22
Non diciamo sbagliata perchè vorrebbe dire che tutto il lavoro fatto sia privo di senso...e non lo è, però l'età può essere imprecisa, potrebbe essere soggetta a variazioni, effetti sistematici che non abbiamo contemplato. E comunque l'età dipende dal modello usato, e se cambi il modello l'età cambia!
Non abbiamo alcun modo di sapere realmente quanto l'Universo sia vecchio, se non estrapolandolo dai modelli.

Ok ;)



Te l'ho già scritto il numero :sneaky:, ti invito a rileggere meglio il mio post.

:confused: ma!? L ho riletto e tu dici "se consideri un raggio di universo osservabile (cioè tutto l'Universo a partire dal Big Bang in sostanza), ottieni un valore di circa 47 miliardi di anni luce. Questo ti da un volume di 4 x 1080*metri cubi."
C è qualcosa che mi sfugge!! Scusa mica diciamo sempre che l universo si estende oltre l universo osservabile? E in questo caso tu mi hai dato il volume dell universo osservabile (la sfera di hubble insomma), ma il volume di TUTTO l universo (anche quello oltre la sfera di hubble) qual è? Non capisco. .

DarknessLight
19-05-2015, 23:12
Ah sì Enrico, semplicemente ci siamo fraintesi: io ti chiedevo il volume dell intero universo (se si conosce), non il volume dell universo osservabile che è semplice da stabilire una volta noto il raggio della sfera di hubble.

Enrico Corsaro
20-05-2015, 01:31
Che io sappia non ci sono stime, purtroppo qui bisognerebbe basarsi su quanto efficace è stata l'accelerazione e questo è un pò difficile da determinare, i dati che abbiamo a disposizione non si spingono così oltre, siamo cioè sempre limitati dalla sfera di Hubble. Diciamo comunque che poichè l'accelerazione ha preso il sopravvento solo in tempi relativamente recenti, il raggio totale non dovrebbe essere molto più grande di quello dell'Universo osservabile. Io direi probabilmente entro un 10% di differenza...se trovo qualche riferimento bibliografico ti faccio sapere.

DarknessLight
20-05-2015, 09:42
So che non ha molto interesse dal punto di vista fisico ma è comunque una curiosità interessante.. a me sembrava di aver letto da qualche parte (forse proprio in un articolo qui nel forum) che l universo oltre la sfera di hubble si estendesse per altri 100 miliardi di anni luce, forse anche molto molto di più... è possibile?
Se invece fosse come dici tu e allora l universo si estende per "solo" un altro 10% allora noi in pratica lo conosciamo quasi tutto...

Red Hanuman
20-05-2015, 10:02
In alcune stime si parla di 93 miliardi di a.l. di diametro, cioè un raggio di 46,5 miliardi di a.l.. Ma si tratta del solo universo osservabile, cioè di quella parte dell'universo che possiamo vedere grazie al fatto che la velocità della luce è limitata, e ciò ci consente di osservare oggetti che ora sono oltre la nostra sfera di Hubble, ma di cui abbiamo immagini di un passato in cui non erano ancora oltre tale limite.... [emoji6]

DarknessLight
20-05-2015, 10:05
Grande!!! Allora avevo capito male io, grazie della spiegazione.
Avevo semplicemente confuso i valori..;)

Morimondo
21-05-2015, 10:29
In alcune stime si parla di 93 miliardi di a.l. di diametro, cioè un raggio di 46,5 miliardi di a.l.. Ma si tratta del solo universo osservabile, cioè di quella parte dell'universo che possiamo vedere grazie al fatto che la velocità della luce è limitata, e ciò ci consente di osservare oggetti che ora sono oltre la nostra sfera di Hubble, ma di cui abbiamo immagini di un passato in cui non erano ancora oltre tale limite.... [emoji6]

Faccio fatica a digerire il principio cosmologico, (per me è quasi come un assioma religioso: ci devi credere punto e basta).
Chiedo se l'universo è in espansione se noi possiamo conoscere una sfera di raggio poco meno di 14 miliardi di anni luce, se comunquue un osservatore sul bordo esterno di questa sfera vedrebbe più o meno quello che vediamo noi grazie al fatto che lo spazio che contiene il tutto si espande a una velocita superiore a quella della luce, a causa dell'ipotesi dell'energia oscura, alla fin fine ci dovrà pur essere un limite esterno: per un osservatore posto a 46,5 miliardi di a.l. vale il principio cosmologico? Guardando in una direzione non vedrà un grande vuoto privo di stelle?
Il principio cosmologico non vale per piccole scale ma solo per regioni di oltre 300 milioni di anni luce ma potrebbe tornare a non valere per scale superiori a 100 miliardi di anni luce?

DarknessLight
21-05-2015, 11:31
Morimondo il principio cosmologico è tutto altro che un dogma religioso: è supportato da innumerevoli osservazioni che confermano l omogeneità e l isotropia dell universo a larga scala. Insomma l universo è come un fluido che si espande e (proprio come per le particelle di un gas in espansione) la materia, la radiazione e le altre componenti si trovano diffuse piu o meno uniformemente per tutto il suo volume.
Esistono poi delle super concentrazioni di materia come il Grande Attrattore, ma pur sempre rientranti in un omogeneità generale: il principio cosmologico resta comunque valido!!!

Per l altra cosa che chiedi (il "grande vuoto" che presumi che esista sui "bordi" dell universo) praticamente tu stai dicendo che l universo ha dei confini: ma non è così!!!
Non esistono limiti o bordi oltre cui non c'è più nulla: il VOLUME dell universo È COME la SUPERFICIE di una sfera: è ILLIMITATO ma NON infinito ;)
Einstein ad esempio parlava dell universo come di una tre sfera (o ipersfera) ovvero un oggetto di 3 dimensioni in uno spazio di 4 dimensioni i cui punti sono equidistanti da un centro.
È normale che tu (e io, e anche chiunque altro compreso Einstein) non riesca ad immaginare una cosa del genere: non abbiamo rappresentazioni mentali in grado di descrivere una 3 sfera, ma ti basti sapere che matematicamente è un oggetto che funziona e che può essere in grado i descrivere l universo.

Pensa a questo: la 2 sfera (quella classica) è un bordo (una superficie) che racchiude un oggetto in tre dimensioni (il volume). La 3 sfera è un bordo a 3 dimensioni (che rappresenta il volume del nostro universo) che racchiude un oggetto in 4 dimensioni, in cui in questo caso la quarta dimensione rappresenta il tempo, ciò significa che il raggio della 3 sfera rappresenta l età dell universo (il tempo cosmico standard) ;)

Questo è quello che ho capito io. Spero di non aver commesso errori.

bertupg
21-05-2015, 12:41
E se l'universo fosse più piccolo di quello che pensiamo? Se la luce emessa da oggetti primordiali avesse avuto il tempo di fare più volte il "giro dell'universo", come l'insetto sulla superficie del palloncino?
Allora magari qualcuno dei quasar che vediamo nelle profondità dello spazio-tempo potrebbe essere un'immagine ancestrale di galassie che vediamo vicino a noi quando erano ancora in via di formazione, se della nostra stessa galassia! Saremmo in grado di riconoscerla?
Mi rendo conto che è una ipotesi più peregrina che fantasiosa, e sicuramente vi sono fior fior di evidenze osservative e sperimentali, oltre che concettuali, che la possono confutare, ma l'ultima domanda potrebbe rimanere valida: se accettassimo questa ipotesi, saremmo in grado di riconoscere, tra tutti gli oggetti più remoti, il precursore della nostra casa, o saremmo come l'abitante di un quartiere periferico di una grande città, che non riconoscerebbe in una sbiadita foto in bianco e nero di 60 anni prima i campi e i prati su cui sono sorti i palazzi a cui è abituato?

davide1334
21-05-2015, 13:48
mettiamo che si stia andando verso il big rip,come pare che sia.la gravità dominerà (sempre più a fatica)solo nelle strutture locali. lo spazio profondo sparirà alla nostra vista,niente più galassie lontane,eccetera. anche la radiazione di fondo raffreddandosi ulteriormente magari non sarà più rilevabile....insomma a quel punto non ci saranno più prove di un big bang? paradossalmente la cosmologia tornerà indietro: avremo una rappresentazione del tutto limitato alla nostra galassia,praticamente come cento anni fa

Arzak
21-05-2015, 14:10
Allora magari qualcuno dei quasar che vediamo nelle profondità dello spazio-tempo potrebbe essere un'immagine ancestrale di galassie che vediamo vicino a noi quando erano ancora in via di formazione, se della nostra stessa galassia! Saremmo in grado di riconoscerla?
L'ipotesi è intrigante, e già l'avevo letta in forma diversa in un vecchio libro di fisica. Si parlava di un ipotetico specchio collocato in un pianeta posta a centinaia anni luce di distanza, e guardando il quale avremmo visto la nostra Terra come era centinaia di anni fa. Si potrebbe quindi immaginare che un raggio di luce, partito dalla nostra galassia alle origini dell'universo, dopo aver fatto qualche giro ricapitasse da noi riportandoci un bel "com'eravamo". Un po' come se qualcuno, dopo averci scattata una foto quando eravamo bambini, ce la consegnasse dopo trent'anni.

L'ipotesi secondo me si potrebbe collegare, sempre in questo ambito di fantasticherie peregrine, a quanto dice @DarknessLight (http://www.astronomia.com/forum/member.php?u=3442) sull'universo illimitato ma non infinito. Si pensi al famoso disegnino di Escher, in cui delle formiche camminano sull'anello di Moebius:


http://www.storiediscienza.it/wp-content/uploads/2013/07/fita_mobius.jpg



Le formichine sono convinte di muoversi in uno spazio a due dimensioni (o addirittura ad una dimensione se camminano solo in avanti) che percorrono in quello che a loro sembra un itinerario infinito. In realtà si trovano in uno spazio a tre dimensioni finito ma illimitato. Si trovano così a percorrere più volte lo stesso itinerario senza accorgersene, così come i raggi di luce nell'ipotesi di @bertupg (http://www.astronomia.com/forum/member.php?u=82).
Estendendo la similitudine si può intuire come noi forse ci troviamo in un universo a 4 dimensioni pur vedendone solo 3. Un po' come le formichine, o come un insetto posto all'interno di una palla cava, che può camminare in tutte le direzioni in uno spazio illimitato apparentemente bidimensionale.

Sulla faccenda che la quarta dimensione sia il tempo, vulgata assai diffusa, ho però forti dubbi. Si usa questa terminologia in riferimento alla posizione di un punto che si muove nello spazio, per determinare la quale occorrono 4 numeri di cui 3 sono le coordinate spaziali e la quarta è appunto il tempo, ma solo perchè siamo interessati all'esclusivo fenomeno cinematico. Se volessimo invece considerare le condizioni di un oggetto in senso più lato potremmo tirare in ballo la sua temperatura, la pressione a cui è sottoposto, l'umidità eccetera. Ma questo non significa che l'oggetto si muova in uno spazio a 7 dimensioni! Si tende insomma a confondere la situazione reale con la sua rappresentazione mediante assi cartesiani, che rimane solo un'artificio grafico-matematico.
La quarta dimensione spaziale, se c'è, deve essere spaziale anch'essa, solo che non possiamo vederla. Non sono in grado di capirne il motivo, solo di intuirlo per analogia con gli esempi di cui sopra. Forse c'entra il fatto che lo spazio è "incurvato" su sé stesso come l'interno della palla cava, o per qualche altra legge relativistica che mi sfugge. E' ben vero che molte di queste cose esulano dalla nostra comprensione, come la massa che si trasforma in energia, ma che il tempo si trasformi in spazio e viceversa non riesco assolutamente a digerirlo, nè mi risulta che vi siano ipotesi o leggi che lo spieghino, dal momento che anche nelle leggi relativistiche più astruse i due termini permangono distinti.
Qui lo dico e qui lo nego.

Red Hanuman
21-05-2015, 22:59
Faccio fatica a digerire il principio cosmologico......

Direi che @DarknessLight (http://www.astronomia.com/forum/member.php?u=3442) ti ha già risposto egregiamente.....


E se l'universo fosse più piccolo di quello che pensiamo? Se la luce emessa da oggetti primordiali avesse avuto il tempo di fare più volte il "giro dell'universo", come l'insetto sulla superficie del palloncino?
Allora magari qualcuno dei quasar che vediamo nelle profondità dello spazio-tempo potrebbe essere un'immagine ancestrale di galassie che vediamo vicino a noi quando erano ancora in via di formazione, se della nostra stessa galassia! Saremmo in grado di riconoscerla?
Mi rendo conto che è una ipotesi più peregrina che fantasiosa, e sicuramente vi sono fior fior di evidenze osservative e sperimentali, oltre che concettuali, che la possono confutare, ma l'ultima domanda potrebbe rimanere valida: se accettassimo questa ipotesi, saremmo in grado di riconoscere, tra tutti gli oggetti più remoti, il precursore della nostra casa, o saremmo come l'abitante di un quartiere periferico di una grande città, che non riconoscerebbe in una sbiadita foto in bianco e nero di 60 anni prima i campi e i prati su cui sono sorti i palazzi a cui è abituato?

Non è un'ipotesi peregrina, anzi! Ne abbiamo già discusso. Prova a guardare tra gli articoli sul portale.... ;)


mettiamo che si stia andando verso il big rip... insomma a quel punto non ci saranno più prove di un big bang? paradossalmente la cosmologia tornerà indietro: avremo una rappresentazione del tutto limitato alla nostra galassia,praticamente come cento anni fa

Hai colto nel segno! Tempo fa, ho letto un articolo su "Le scienze" che diceva le stesse cose che dici tu... Purtroppo, è una conseguenza inevitabile dell'accelerazione dell'espansione dello spazio... :mad:

Enrico Corsaro
22-05-2015, 06:28
@Morimondo (http://www.astronomia.com/forum/member.php?u=3620), in verità, si fa sempre più fatica a digerire il principio cosmologico proprio perchè scopriamo una quantità di dettagli nelle strutture che ci fanno capire che l'Universo è tutt'altro che omogeneo ed isotropo...tuttavia, diciamo che su scale di grandezza molto grandi, cosmologiche appunto, dell'ordine del centinaio di milioni di anni luce, il principio cosmologico è pressochè accettabile...


Riguardo alla possibilità di avere segnali emessi da noi che potrebbero essere in grado di ritornare...faccio a voi un paio di domande:
1) Considerato che le distanze che osserviamo sono in allontanamento e che oltre un certo limite l'espansione supera la velocità della luce, pensate davvero che un segnale possa essere rivelato dopo aver viaggiato attraverso tutto l'Universo?
2) Se il percorso dei raggi luminosi dovesse essere chiuso, come risulterebbe dall'assumere che la luce ritorni nello stesso punto da cui è stata emessa dopo un certo intervallo di tempo, significa che la geometria non è Euclidea, ma sferica. Sappiamo però dai dati (anche indipendenti dal modello cosmologico) che la geometria dello spazio-tempo è proprio Euclidea. Questo vi dice qualcosa?

Riguardo alle dimensioni citate da @Arzak (http://www.astronomia.com/forum/member.php?u=3374), non sono d'accordo. Il tempo non è una funzione di stato, come invece lo sono pressione, densità, ecc., nè è una proprietà fisica di un oggetto. Il tempo è un riferimento, così come lo sono le coordinate spaziali, ed è proprio qui il punto sostanziale che lo differenzia! Potreste cambiare il riferimento del tempo semplicemente cambiando il punto iniziale a cui lo considerate. Inoltre il tempo non ha di per se una caratterizzazione fisica, cioè non è una proprietà di qualcosa, di un oggetto. Parametri fisici invece come la densità, la pressione, la temperatura, sono caratteristiche ben precise di un corpo e che non cambiano al cambiare del riferimento che utilizziamo (possono solo cambiare i valori numerici che hanno se si cambia il sistema di unità di misura).
La coordinata è un concetto di base astratto per l'appunto e può essere definita nel modo che reputiamo più opportuno, in base al modo in cui conviene di più a noi. Non è così invece per le proprietà fisiche di cui sopra, che sono realmente legate alle caratteristiche del corpo in oggetto.
Non c'è confusione di rappresentazione qui, sono due cose distinte e separate. Per esprimere numericamente le proprietà fisiche di un oggetto, qualsiasi esse siano, utilizzi un sistema di riferimento (astratto) e quel sistema di riferimento è costituito da delle coordinate. Nell'Universo in cui viviamo, questo sistema di riferimento ha 4 coordinate, di fatto non ne servono di più per creare un riferimento su ciò che ci sta intorno. Siamo noi che percepiamo le 4 dimensioni e di conseguenza il sistema di riferimento nasce in 4 dimensioni.

L'errore che state facendo adesso secondo me è pensare che il palloncino con la sua superficie è inserito in uno spazio che sappiamo essere tridimensionale, e sappiamo che tale spazio esiste tra il centro del palloncino e la sua superficie. Dunque si proietta lo stesso con l'Universo e quindi si tende a credere che ci sia "spazio" tra il centro della ipersuperficie in 3D e la ipersuperficie stessa dell'Universo. Non è così che funziona il confronto...
Se pensate al palloncino, dovete pensare unicamente alla sua superficie e dimenticarvi del resto, come se non esistesse, come se voi stessi viveste in un mondo in due sole dimensioni.
Non c'è motivo di introdurre una dimensione ulteriore se tutto ciò che avviene è confinato sulla superficie stessa. Nel qual caso, non c'è motivo di introdurre una quarta dimensione spaziale se tutto ciò che percepiamo e viviamo è in 3 dimensioni spaziali. E' un errore comune e richiede un pò di riflessione per essere assimilato...

Il fatto che noi visualizziamo un nastro di Moebius che di per sè è una superficie in 2D dislocarsi in uno spazio in 3D, non significa che un ipotetico nastro di Moebius in 3D sia situato dentro uno spazio in 4D. All'atto pratico quello che consideriamo è che oltre l'Universo non ha senso considerare l'esistenza di qualcosa. Di conseguenza non ha senso dire che il nostro Universo è una superficie in 3D in uno spazio in 4D (inteso come sole coordinate spaziali!).
Il disegno di Escher che Arzak ha indicato è altamente fuorviante. Perchè? Perchè sul nastro che è una superificie in 2D sono disegnate delle formiche in 3D, il che fa cadere in errore. Se dovessimo essere consistenti, le formiche dovrebbero essere disegnate non in 3D ma in 2D, cioè appiattite sulla superficie del nastro. Ci dovremmo allora immaginare di essere noi stessi appiattiti allo stesso modo. Allora per noi la terza dimensione semplicemente non esisterebbe. Se volete potete anche pensare al mondo di Flatlandia di Edwin Abbott.

Come diceva Hawking, chiedersi cosa c'è al di là del Big Bang (quindi se volete "fuori" dall'Universo) è un pò come chiedersi cosa c'è sulla superficie della Terra un pò più a Nord del Polo Nord. Per l'Universo questa domanda non ha senso.

Red Hanuman
22-05-2015, 07:35
Riguardo alla possibilità di avere segnali emessi da noi che potrebbero essere in grado di ritornare...faccio a voi un paio di domande:
1) Considerato che le distanze che osserviamo sono in allontanamento e che oltre un certo limite l'espansione supera la velocità della luce, pensate davvero che un segnale possa essere rivelato dopo aver viaggiato attraverso tutto l'Universo?
2) Se il percorso dei raggi luminosi dovesse essere chiuso, come risulterebbe dall'assumere che la luce ritorni nello stesso punto da cui è stata emessa dopo un certo intervallo di tempo, significa che la geometria non è Euclidea, ma sferica. Sappiamo però dai dati (anche indipendenti dal modello cosmologico) che la geometria dello spazio-tempo è proprio Euclidea. Questo vi dice qualcosa?

Una riedizione del paradosso di Olbers (http://it.wikipedia.org/wiki/Paradosso_di_Olbers).... Ne abbiamo già parlato, e ci sono degli articoli sul tema nel portale...;)

Enrico Corsaro
22-05-2015, 07:41
Una riedizione del paradosso di Olbers (http://it.wikipedia.org/wiki/Paradosso_di_Olbers).... Ne abbiamo già parlato, e ci sono degli articoli sul tema nel portale...;)

Si esattamente, le mie erano domande provocatorie per far capire che la risposta la sappiamo già ;).

bertupg
22-05-2015, 09:36
significa che la geometria non è Euclidea, ma sferica. Sappiamo però dai dati (anche indipendenti dal modello cosmologico) che la geometria dello spazio-tempo è proprio Euclidea.

Su tutto il resto sono perfettamente d'accordo.
Tuttavia su questo punto continuo ad avere dei dubbi. Ok, prendiamo per assodato che la geometria sia euclidea, ma mi sembra che sia dato per altrettanto assodato che sia finito e illimitato, come ha giustamente scirtto DarknessLight nel suo post, al quale il mio appunto si ricollegava.

Ora, io non sono un esperto, ma da quello che mi ricordo mi sembra che esistano spazi geometrici che possono soddisfare contemporaneamente queste caratteristiche (a me viene in mente il toro ma mi sembrava che ce ne fossero anche altri).

La domanda era se sia vero che se potessimo viaggiare a velocità infinita e scegliessimo una particolare direzione, prima o poi potremmo tornare al punto di partenza. Questo era per consentirmi di visualizzare in modo intuitivo come possa accadere che non esiste un "vuoto oltre i confini dell'universo".

Purtroppo scrivendo mi sono lasciato trasportare e alla fine mi sono focalizzato più sulla ipotetica possibilità di vedere il nostro stesso passato che sul concetto astratto di spazio euclideo finito e illimitato.
Chiedo scusa se questo ha creato più confusione di quella che avrebbe voluto risolvere.

DarknessLight
22-05-2015, 12:07
@Morimondo (http://www.astronomia.com/forum/member.php?u=3620), in verità, si fa sempre più fatica a digerire il principio cosmologico proprio perchè scopriamo una quantità di dettagli nelle strutture che ci fanno capire che l'Universo è tutt'altro che omogeneo ed isotropo...tuttavia, diciamo che su scale di grandezza molto grandi, cosmologiche appunto, dell'ordine del centinaio di milioni di anni luce, il principio cosmologico è pressochè accettabile...

:meh: :confused:
D accordo può essere che su piccole scale si possano trovare delle fluttuazioni (come dicevo il Grande Attrattore) che sembrano discordare con il Principio cosmologico, ma mi ricordo che tu stesso una volta mi avevi detto che queste strutture sono piccola cosa rispetto le dimensioni cosmiche e perciò il principio resta validissimo.
Come mai ora dici che si fa sempre più fatica a digerire il principio cosmologico? Addirittura dici che l universo è tutto altro che omogeneo e isotropo.. ma?! È molto strano che l universo non sia omogeneo ed isotropo... significherebbe che esistono direzioni preferenziali, o meglio che la simmetria non esiste... cosa ne pensi?


Se il percorso dei raggi luminosi dovesse essere chiuso, come risulterebbe dall'assumere che la luce ritorni nello stesso punto da cui è stata emessa dopo un certo intervallo di tempo, significa che la geometria non è Euclidea, ma sferica

ok parliamo di geometria Euclidea: l universo è comunque finito e illimitato, come la superficie di una sfera, quindi in TEORIA la luce dovrebbe tornare nel punto da cui è partita. Sono d accordo con bertupg
Io credevo questo: la geometria Euclidea fa viaggiare il fotone in linea retta, le altre invece lo fanno curare negativamente o positivamente, ma ciò NON influisce con il fatto che l universo sia un superficie chiusa e che Perciò l informazione possa tornare ESATTAMENTE al punto di partenza.


L''errore che state facendo adesso secondo me è pensare che il palloncino con la sua superficie è inserito in uno spazio che sappiamo essere tridimensionale, e sappiamo che tale spazio esiste tra il centro del palloncino e la sua superficie. Dunque si proietta lo stesso con l'Universo e quindi si tende a credere che ci sia "spazio" tra il centro della ipersuperficie in 3D e la ipersuperficie stessa dell'Universo. Non è così che funziona il confronto...
Se pensate al palloncino, dovete pensare unicamente alla sua superficie e dimenticarvi del resto, come se non esistesse, come se voi stessi viveste in un mondo in due sole dimensioni.

Ok, ma tutto il discorso che fai riguardo le dimensioni in risposta ad Arzak, mi chiedo se sia riferito anche a questa mia spiegazione data in precedenza a Morimondo: "il VOLUME dell universo È COME la SUPERFICIE di una sfera: è ILLIMITATO ma NON infinito. Einstein ad esempio parlava dell universo come di una tre sfera (o ipersfera) ovvero un oggetto di 3 dimensioni in uno spazio di 4 dimensioni i cui punti sono equidistanti da un centro. La 2 sfera (quella classica) è un bordo (una superficie) che racchiude un oggetto in tre dimensioni (il volume). La 3 sfera è un bordo a 3 dimensioni (che rappresenta il volume del nostro universo) che racchiude un oggetto in 4 dimensioni, in cui in questo caso la quarta dimensione rappresenta il tempo, ciò significa che il raggio della 3 sfera rappresenta l età dell universo (il tempo cosmico standard)."

A me pare che sia una spiegazione corretta e coerente. La quarta dimensione temporale è necessaria per spiegare l universo. Credo che il volume dell universo sia la ipersuperficie della 3 sfera, e che il raggio della 3 sfera rappresenti in qualche modo il tempo cosmico standard, insomma, per analogia, l ipersuperficie è il volume del nostro universo, e l ipervolume è la dimensione spazio-temporale ovvero tempo + spazio, cioè il tutto.
cosa ne dici?

givi
22-05-2015, 17:40
Splendida discussione

Arzak
22-05-2015, 19:25
Il disegno di Escher che Arzak ha indicato è altamente fuorviante. Perchè? Perchè sul nastro che è una superificie in 2D sono disegnate delle formiche in 3D, il che fa cadere in errore. Se dovessimo essere consistenti, le formiche dovrebbero essere disegnate non in 3D ma in 2D, cioè appiattite sulla superficie del nastro. Ci dovremmo allora immaginare di essere noi stessi appiattiti allo stesso modo. Allora per noi la terza dimensione semplicemente non esisterebbe. Se volete potete anche pensare al mondo di Flatlandia di Edwin Abbott.

Certo, ma la colpa è di Escher che è un disegnatore, e non un fisico. Un vero fisico come Carlson, autore di un libro di fisica degli anni 30 che ho trovato in casa, divulgativo ma estremamente chiaro nelle sue spiegazioni idiot-proof, nel citare un esempio simile parla di cimici, e non di formiche, immaginandole quindi bidimensionali. Vedi come l'entomologia aiuta la cosmologia...

L'idea della quarta dimensione, che vedo serpeggiare qua e là anche in questo forum, e che alcuni intendono spaziale ed altri temporale, ho cercato di spiegarmela estrapolando il discorso bidimensionale di Moebius. Se poi non esiste ce ne faremo una ragione. Ciò che mi sembrava discutibile era l'attribuzione allo spazio reale della rappresentazione cartesiana di esso, come se il tempo fosse davvero la quarta dimensione spaziale, e non un semplice parametro utile a localizzare un punto nello spazio. Questo almeno ho creduto di interpretare in qualche intervento.

DarknessLight
22-05-2015, 19:41
Ciò che mi sembrava discutibile era l'attribuzione allo spazio reale della rappresentazione cartesiana di esso, come se il tempo fosse davvero la quarta dimensione spaziale, e non un semplice parametro utile a localizzare un punto nello spazio. Questo almeno ho creduto di interpretare in qualche intervento.

Io credo proprio che la spiegazione ultima della realtà possa essere compiuta unicamente attraverso la matematica. La fisica e a maggior ragione le altre discipline sono solo nostre approssimazioni, sono modelli tipicamente umani con cui descrivere i fenomeni attraverso il filtro della mente...
Perciò direi proprio che spazio, tempo, energia, massa, temperatura, ecc.. sono solo nostre descrizioni della realtà... e solo noi umani le possiamo utilizzare, ma altri esseri ne usano di differenti.... solo la matematica é assoluta...
quello che noi definiamo spazio e tempo sono in realta coordinate all interno di uno spazio quadrimensionale.... tra loro non vi è una reale differenza...
È l uso funzionale che ne facciamo che le rende tali da essere interpretate in maniera diversa...
Insomma, il cervello è un computer che lavora per archetipi, è ovvio che noi ragioniamo in termini di spazio, tempo, massa, velocità... ma la realtà è ben altra... e solo la pura astrazione matematica può essere in grado di descriverla in maniera fondamentale!!!

Enrico Corsaro
22-05-2015, 19:45
Ragazzi seguo con un pò di fatica tutti i post nel forum perchè scrivete in tanti e le discussioni sono molte...cerco di rispondere appena possibile alle domande, ma se vedete che mi dimentico per più giorni in caso fatemi un tag!

DarknessLight
22-05-2015, 19:48
Enrico vai tranquillo... quello che fai è già troppo..senza fretta mettici tutto il tempo che ti serve ;)

Arzak
22-05-2015, 23:06
Io credo proprio che la spiegazione ultima della realtà possa essere compiuta unicamente attraverso la matematica. La fisica e a maggior ragione le altre discipline sono solo nostre approssimazioni, sono modelli tipicamente umani con cui descrivere i fenomeni attraverso il filtro della mente...


Hai aperto un problema epistemologico assai interessante, ma non so se sia il caso di approfondirlo. Non lo dico per me, ma per i lettori che forse si distrarrebbero. Rispondendo in breve, non sarei così categorico. Sia la fisica che la matematica contribuiscono in modo inscindibile alla conoscenza del mondo. Si potrebbe dire che la prima è il braccio, la seconda la mente, oppure il motore e la benzina, o la sostanza e la forma, o la chora e le idee, o il mondo reale e le monadi, o la tesi e l'antitesi, o la musica e il pentagramma, o la tavolozza ed il colore, o la Ram e il microprocessore, o la pastasciutta e il sugo, o Watson e Sherlock Holmes, o Pippo e Topolino....

continuo? :biggrin:

DarknessLight
22-05-2015, 23:24
Sì sì non divaghiamo, però volevo mettere in luce il fatto che determinate categorie sono prettamente umane, ma alla fin fine tutto può essere sintetizzato in termini assolutamente oggettivi, per questo esistono le formalizzazioni matematiche. Come dire, il tempo è una nostra percezione mentale, e io credo che alla fin fine rappresenti semplicemente il nostro modo per interpretare un fenomeno: nello specifico l aumento di entropia dell universo, infatti se noti la percezione della dimensione temporale nasce dai fenomeni di dissipazione di calore. È grazie all entropia che noi definiamo una freccia temporale.. da ciò mi pare logico pensare che in fondo spazio e tempo sono solo modi umani per vedere delle coordinate cartesiane...
Però in effetti è meglio non divagare. Chiudiamo qui e ne parliamo in una discussione a parte ;)

Arzak
22-05-2015, 23:43
Sse noti la percezione della dimensione temporale nasce dai fenomeni di dissipazione di calore. È grazie all entropia che noi definiamo una freccia temporale.. da ciò mi pare logico pensare che in fondo spazio e tempo sono solo modi umani per vedere delle coordinate cartesiane...
Però in effetti è meglio non divagare. Chiudiamo qui e ne parliamo in una discussione a parte ;)

Anche questo è uno spunto assai intrigante e per me inedito. Non so se sia del tutto condivisibile, comunque se apri un altro thread sul tema cercherò di dire la mia.

(basta che per Natale prometti di comprarti una tastiera nuova dotata di apostrofi... http://r15.imgfast.net/users/1517/32/07/50/smiles/977590602.gif)
http://r15.imgfast.net/users/1517/32/07/50/smiles/2007200199.gif

DarknessLight
23-05-2015, 00:14
Arzak non so proprio in che sezione aprire questo thread. Comunque (dico l ultimissima cosa poi davvero è meglio chiudere qui: davvero vorrei continuare ma siamo già troppo fuori tema) sul discorso dell entropia che introduce una freccia temporale ne sono abbastanza sicuro. Ma davvero ti suona nuovo? Mi sembra strano..
Infatti pensa a questo esempio (preso dal libro di Feymann: non ricordo se fosse "sei pezzi facili" o "sei pezzi meno facili"): rompo un bicchiere sul pavimento. Poi decido di calcolare ogni singola forza che ha agito su ogni singola particella del bicchiere durante la frantumazione. Ebbene noto che ogni forza a livello sub atomico è perfettamente reversibile.
Ora analizzo il fenomeno nella sua interezza e noto che durante la frantumazione del bicchiere è avvenuto un fenomeno entropico ovvero uno scambio di calore tra sistema e ambiente che si traduce con un aumento del disordine generale.
ebbene, per le particelle capisci che le forze che hanno agito, essendo reversibili, non mostrano una direzione preferenziale e quindi non indicano una freccia del tempo, poiché l inizio coincide con la fine. Quindi il tempo non sussiste.
Invece nel fenomeno preso nella sua interezza posso definire un momento iniziale in cui l entropia è bassa e un momento finale in cui l entropia è più alta. Quindi posso indicare una direzione preferenziale. Quindi posso indicare una freccia temporale.
Sulla tomba di Bolzmann è incisa la formula dell entropia S=k logW. Significa che l entropia rappresenta un numero maggiore di stadi in cui si può trovare un sistema: per capirci, esistono più modi per avere un bicchiere rotto che per avere un bicchiere aggiustato.
Comunque sia quello che voglio dire è che nei fenomeni puramente meccanici (che ovviamente non esistono se non in termini ideali) non esiste una direzione preferenziale in cui il fenomeno debba evolversi poiché non vi è variazione di entropia. Diverso è nei fenomeni termodinamica che invece indicano due differenti situazioni, quindi due differenti stadi in cui possiamo trovare un sistema: da qui nascono il concetto di "prima" e "dopo" ;)

Arzak
23-05-2015, 00:23
Ma davvero ti suona nuovo? Mi sembra strano..

Beh, non sono un fisico, tanto per iniziare. Comunque, so dell'entropia, entalpia e compagnia cantante, ma non l'avevo mai associata alla misurazione del tempo. Per il Thread, se vuoi lo apriamo al bar, così anche se ci scappa qualche discorso da ubriachi nessuno ci farà caso. Dimmi tu.

DarknessLight
23-05-2015, 00:23
basta che per Natale prometti di comprarti una tastiera nuova dotata di apostrofi...

In realtà gli apostrofi ci sono, ma sono scritti con inchiostro invisibile... prova a mettere lo schermo sopra una fiamma e li vedrai comparire ;)

DarknessLight
23-05-2015, 00:28
Se vuoi aprine uno al bar e poi taggami. Ora però ti faccio la buona notte perchè per me it's nanna time ;)

Marcos64
23-05-2015, 11:13
.....Per il Thread, se vuoi lo apriamo al bar, così anche se ci scappa qualche discorso da ubriachi nessuno ci farà caso. Dimmi tu.
Ottima idea! :) ...ma poi non fate come quelli che stanno tutto il giorno al bar giocando a carte, e si
bevono quando va bene solo un caffe'!

Enrico Corsaro
25-05-2015, 02:44
Ora, io non sono un esperto, ma da quello che mi ricordo mi sembra che esistano spazi geometrici che possono soddisfare contemporaneamente queste caratteristiche (a me viene in mente il toro ma mi sembrava che ce ne fossero anche altri).

La domanda era se sia vero che se potessimo viaggiare a velocità infinita e scegliessimo una particolare direzione, prima o poi potremmo tornare al punto di partenza. Questo era per consentirmi di visualizzare in modo intuitivo come possa accadere che non esiste un "vuoto oltre i confini dell'universo".

Ho provato un pò a rifletterci, e la risposta che mi sono dato è la seguente: siamo sempre tentati di pensare all'Universo come un oggetto in 3D, con distanze ben fissate e immutabili. In realtà l'Universo è concepito come una varietà riemanniana in 4D, che è tutt'altro che un semplice spazio euclideo in 3 dimensioni. Infatti, ciò che dobbiamo considerare è che un percorso di luce non può mai ritornare nello stesso punto di partenza per un motivo fondamentale, che l'Universo contiene anche la quarta dimensione, il tempo, che non rende mai due istanti identici l'uno all'altro (se così fosse l'Universo sarebbe statico, ma sappiamo che non lo è dall'evidenza osservativa). Credo che dunque il tempo, cioè la quarta dimensione, rende di fatto l'Universo impossibile da "ripercorrere" in ogni suo punto. Ed è proprio al tempo che associamo infatti l'evoluzione del nostro Universo (che sia espansione o contrazione, ecc.). Porci la stessa domanda per un ipotetico Universo in 3D soltanto sostanzialmente non ha senso, perchè non è ciò di cui noi facciamo esperienza e perchè i segnali luminosi stessi per propagarsi hanno bisogno di tempo. In un Universo in 3D (cioè puramente statico), non avrebbe dunque neanche senso parlare di un segnale che si propaga.
Secondo me questa interpretazione è plausibile e personalmente mi da senso perchè in accordo con quanto la RG ci spiega.

Enrico Corsaro
25-05-2015, 03:00
:meh: :confused:
D accordo può essere che su piccole scale si possano trovare delle fluttuazioni (come dicevo il Grande Attrattore) che sembrano discordare con il Principio cosmologico, ma mi ricordo che tu stesso una volta mi avevi detto che queste strutture sono piccola cosa rispetto le dimensioni cosmiche e perciò il principio resta validissimo.
Come mai ora dici che si fa sempre più fatica a digerire il principio cosmologico? Addirittura dici che l universo è tutto altro che omogeneo e isotropo.. ma?! È molto strano che l universo non sia omogeneo ed isotropo... significherebbe che esistono direzioni preferenziali, o meglio che la simmetria non esiste... cosa ne pensi?

Perchè man mano gli strumenti diventano più potenti, e ci forniscono visioni più dettagliate e complete delle strutture cosmiche a larga scala. Conta che il principio cosmologico è stato formulato negli anni '30. Tuttavia se parliamo comunque di evoluzione GLOBALE dell'Universo, può rimanere ancora una approssimazione valida, ma ti assicuro che c'è un pò di preoccupazione in tal senso.
Il fatto che non sia omogeneo ed isotropo (e lo vediamo bene già dalla CBR) può essere semplicemente una questione statistica. Le fluttuazioni quantistiche dell'Universo primordiale hanno prodotto determinate variazioni, più o meno alte, nella densità, che poi si sono proiettate su larga scala a causa dell'inflazione. Può accadere che il risultato non sia di fatto perfettamente equilibrato tra massimi e minimi, semplicemente per una questione di casualità. E' una cosa abbastanza normale in ambito fisico. La perfezione non esiste, nè l'equilibrio perfetto dunque.



ok parliamo di geometria Euclidea: l universo è comunque finito e illimitato, come la superficie di una sfera, quindi in TEORIA la luce dovrebbe tornare nel punto da cui è partita. Sono d accordo con @bertupg (http://www.astronomia.com/forum/member.php?u=82)
Io credevo questo: la geometria Euclidea fa viaggiare il fotone in linea retta, le altre invece lo fanno curare negativamente o positivamente, ma ciò NON influisce con il fatto che l universo sia un superficie chiusa e che Perciò l informazione possa tornare ESATTAMENTE al punto di partenza.

Si ho spiegato la mia risposta a questo proprio nel post precedente. In questo caso è il tempo che fa la differenza, e non dobbiamo pensare all'Universo solo come ad un oggetto in 3D, questo induce in errore.



Ok, ma tutto il discorso che fai riguardo le dimensioni in risposta ad Arzak, mi chiedo se sia riferito anche a questa mia spiegazione data in precedenza a Morimondo: "il VOLUME dell universo È COME la SUPERFICIE di una sfera: è ILLIMITATO ma NON infinito. Einstein ad esempio parlava dell universo come di una tre sfera (o ipersfera) ovvero un oggetto di 3 dimensioni in uno spazio di 4 dimensioni i cui punti sono equidistanti da un centro. La 2 sfera (quella classica) è un bordo (una superficie) che racchiude un oggetto in tre dimensioni (il volume). La 3 sfera è un bordo a 3 dimensioni (che rappresenta il volume del nostro universo) che racchiude un oggetto in 4 dimensioni, in cui in questo caso la quarta dimensione rappresenta il tempo, ciò significa che il raggio della 3 sfera rappresenta l età dell universo (il tempo cosmico standard)."

In verità non sono molto d'accordo, perchè il tempo è qualcosa di connesso a tutte e tre le coordinate spaziali e che ne rappresenta la loro evoluzione. Quindi non è solo legato al "raggio" di cui parli, ma più intimamente a tutte le tre coordinate spaziali. Inoltre, mi viene abbastanza strano immaginare che tre coordinate spaziali racchiudono quella temporale che rappresenta un raggio, non trovi sia un pò discordante?
Il raggio è una coordinata spaziale. Il tempo invece ti da l'evoluzione del raggio.
Direi di accantonare l'analogia palloncino in 4D perchè francamente mi sembra fuorviante e non l'ho mai sentita da nessuna parte tra le altre cose.
L'unico scopo del caro palloncino è quello di illustrarci una rappresentazione in 2D dell'espansione dell'Universo (che di fatto avviene in 4D)...



La quarta dimensione temporale è necessaria per spiegare l universo. Credo che il volume dell universo sia la ipersuperficie della 3 sfera, e che il raggio della 3 sfera rappresenti in qualche modo il tempo cosmico standard, insomma, per analogia, l ipersuperficie è il volume del nostro universo, e l ipervolume è la dimensione spazio-temporale ovvero tempo + spazio, cioè il tutto.
cosa ne dici?
Se parli di 3-sfera, il volume che indichi è il volume spaziale. Se vuoi considerare un volume in modo rigoroso dovresti considerare il volume in 4D, cioè un volume globale che dipende anche dal tempo, oltre che dalle tre coordinate spaziali, che tra le altre cose va fuori dal nostro modo di concepire un volume perchè possiamo solo immaginarlo a 3 dimensioni alla volta.

DarknessLight
25-05-2015, 12:18
Mettici tutto i tempo che ti occorre per rispondermi. Io ti scrivo la domanda se no poi mi dimentico ma tu fai pure con calma ;)


Il fatto che non sia omogeneo ed isotropo (e lo vediamo bene già dalla CBR) può essere semplicemente una questione statistica. Le fluttuazioni quantistiche dell'Universo primordiale hanno prodotto determinate variazioni, più o meno alte, nella densità, che poi si sono proiettate su larga scala a causa dell'inflazione. Può accadere che il risultato non sia di fatto perfettamente equilibrato tra massimi e minimi, semplicemente per una questione di casualità. E' una cosa abbastanza normale in ambito fisico. La perfezione non esiste, nè l'equilibrio perfetto dunque

Posso capire che delle fluttuazioni quantistiche iniziali si siano poi espanse e abbiano creato maxi strutture e che l universo appare non perfettamente omogeneo. Lo stesso avviene infatti anche in un gas in espansione dove in linea teorica tutte le particelle dovrebbero distribuirsi uniformemente nel volume anche se poi è ovvio che ci sono zone in pò più dense e zone un pò meno dense.
Suppongo che sia questa la risposta, ma il principio cosmologico dovrebbe restare perfettamente valido. Voglio dire, oltre alle fluttuazioni quantistiche cosa potrebbe aver generato le disomogeneità di cui parli?
Forse l universo è ancora troppo giovane e troppo denso per mostrarci una perfetta omogeneità, ma se il principio cosmologico venisse meno significherebbe che esistono delle non simmetrie nelle forze...
tra l altro il principio cosmologico si applica proprio alla legge di hubble e afferma che ogni punto di osservazione è equivalente. Se non fosse così significherebbe che esistono punti di osservazione preferenziali...
Senza il principio cosmologico non si dovrebbe rivedere tutta la cosmologia? Forse tutta la fisica delle forze fondamentali?


Si ho spiegato la mia risposta a questo proprio nel post precedente. In questo caso è il tempo che fa la differenza, e non dobbiamo pensare all'Universo solo come ad un oggetto in 3D, questo induce in errore.

Si capisco cosa vuoi dire, cioè che con la 4 dimensione siamo soggetti ad un espansione e quindi il fotone non tornerà mai nello stesso punto da cui è partito.
forse l esempio è mal posto, ma ammettendo che non ci sia espansione il fotone (per una geometria Euclidea) non dovrebbe comunque tornare nel punto di partenza?
Cioè la domanda vera è: è giusto dire che l universo è finito ma illimitato come la superficie di una sfera e che quindi non ha bordi o margini bensì è tutto collegato e si può ritornare al punto di partenza solo viaggiando in linea retta? (Ammettendo che l espansione non esista e che lo spazio si mantenga uguale a se stesso, ecc. .)
Io ho sempre creduto che fosse così...


In verità non sono molto d'accordo, perchè il tempo è qualcosa di connesso a tutte e tre le coordinate spaziali e che ne rappresenta la loro evoluzione. Quindi non è solo legato al "raggio" di cui parli, ma più intimamente a tutte le tre coordinate spaziali. Inoltre, mi viene abbastanza strano immaginare che tre coordinate spaziali racchiudono quella temporale che rappresenta un raggio, non trovi sia un pò discordante?
Il raggio è una coordinata spaziale. Il tempo invece ti da l'evoluzione del raggio

No aspetta, scusa un attimo, a parte che io stavo parlando di una 3 sfera e non di un palloncino (2 sfera), ma poi la connessione del tempo a mio modo di vedere c è eccome in questo esempio.
La 3 sfera è la superficie tridimensionale di un oggetto in uno spazio quadrimensionale, proprio come il nostro universo, dove la quarta dimensione (il "raggio" della 3sfera) rappresenta il tempo dell universo.
Avevo letto degli articoli di Vincenzo Zappalà in cui veniva fatta questa analogia ma invece di parlare di 3-sfera si parlava di cerchio che rappresentava il 3D dell universo (il volume) e il raggio del cerchio che via via aumentava rappresentava il tempo e quindi più aumenta il raggio (tempo che trascorre) più aumenta la circonferenza (volume che si espande).
Io ho semplicemente esteso il concetto alla 3 sfera e personalmente l analogia mi sembra ottima... non capisco dove sia il mio errore...
Non ho ancora capito come riportare in un commento il link di un articolo :oops: però se mi dici come si fa ti riporto l articolo di Zappalà così vedi tu stesso e poi mi dici.


Se parli di 3-sfera, il volume che indichi è il volume spaziale. Se vuoi considerare un volume in modo rigoroso dovresti considerare il volume in 4D, cioè un volume globale che dipende anche dal tempo, oltre che dalle tre coordinate spaziali, che tra le altre cose va fuori dal nostro modo di concepire un volume perchè possiamo solo immaginarlo a 3 dimensioni alla volta.

E qui è lo stesso di prima.
La quarta dimensione della 3sfera coincide con la quarta dimensione del nostro universo..
Ma io ho anche letto un libro di Rovelli in cui mette in bocca ad Einstein questo discorso ovvero che l universo è una 3sfera e a me sembra un analogia funzionante..
Ovviamente se tu mi dici che non è così io mi fido, ma devo capire anche il perché secondo te è sbagliata.. a me continuano a sembrare giusta...

Enrico Corsaro
26-05-2015, 06:44
Suppongo che sia questa la risposta, ma il principio cosmologico dovrebbe restare perfettamente valido. Voglio dire, oltre alle fluttuazioni quantistiche cosa potrebbe aver generato le disomogeneità di cui parli?
Forse l universo è ancora troppo giovane e troppo denso per mostrarci una perfetta omogeneità, ma se il principio cosmologico venisse meno significherebbe che esistono delle non simmetrie nelle forze...
tra l altro il principio cosmologico si applica proprio alla legge di hubble e afferma che ogni punto di osservazione è equivalente. Se non fosse così significherebbe che esistono punti di osservazione preferenziali...
Senza il principio cosmologico non si dovrebbe rivedere tutta la cosmologia? Forse tutta la fisica delle forze fondamentali?

Giusto giusto certamente. Se ancora viene tenuto in vita significa che effettivamente va bene, ma ricorda, è sempre una approssimazione. Niente è perfetto qui, nè tantomeno l'omogeneità e l'isotropia.
Nulla toglie che in futuro il principio cosmologico possa essere rimpiazzato da una distribuzione effettiva della materia dentro l'Universo, in funzione delle 4 coordinate...per il momento ci teniamo il principio cosmologico.



Si capisco cosa vuoi dire, cioè che con la 4 dimensione siamo soggetti ad un espansione e quindi il fotone non tornerà mai nello stesso punto da cui è partito.
forse l esempio è mal posto, ma ammettendo che non ci sia espansione il fotone (per una geometria Euclidea) non dovrebbe comunque tornare nel punto di partenza?
Cioè la domanda vera è: è giusto dire che l universo è finito ma illimitato come la superficie di una sfera e che quindi non ha bordi o margini bensì è tutto collegato e si può ritornare al punto di partenza solo viaggiando in linea retta? (Ammettendo che l espansione non esista e che lo spazio si mantenga uguale a se stesso, ecc. .)
Io ho sempre creduto che fosse così...

Capisco dove vuoi andare a parare, ma purtroppo non ha senso porsi la domanda in questi termini, nè cercare di immaginarsi una soluzione del genere, quando sappiamo già che le cose funzionano diversamente.
La risposta corretta è, l'universo è un oggetto in 4 dimensioni (che non possiamo visualizzare) e i raggi luminosi non torneranno mai al punto di prima anche in un tempo infinito, perchè l'Universo non è un oggetto statico, si evolve cioè in funzione del tempo, una delle quattro coordinate.



No aspetta, scusa un attimo, a parte che io stavo parlando di una 3 sfera e non di un palloncino (2 sfera), ma poi la connessione del tempo a mio modo di vedere c è eccome in questo esempio.
La 3 sfera è la superficie tridimensionale di un oggetto in uno spazio quadrimensionale, proprio come il nostro universo, dove la quarta dimensione (il "raggio" della 3sfera) rappresenta il tempo dell universo.
Avevo letto degli articoli di Vincenzo Zappalà in cui veniva fatta questa analogia ma invece di parlare di 3-sfera si parlava di cerchio che rappresentava il 3D dell universo (il volume) e il raggio del cerchio che via via aumentava rappresentava il tempo e quindi più aumenta il raggio (tempo che trascorre) più aumenta la circonferenza (volume che si espande).

Secondo me è fuorviante questa visione, abbi pazienza, perchè quello che fai è immaginare la coordinata temporale come una dimensione spaziale, quando invece non puoi visualizzarla. Io personalmente mi astengo dal prenderla per buona. Se poi la vuoi usare come esercizio mentale per immaginarti questo universo in 4 dimensioni OK, non c'è problema, purchè rimani consapevole del fatto che NON puoi visualizzarlo realmente. Puoi immaginare una 3-sfera spaziale, in uno spazio a 4 dimensioni spaziali, ed è una visione coerente, ma non in uno spazio-tempo a 4 dimensioni.
Quello che puoi fare è invece immaginare uno spazio a 3 dimensioni per un istante di tempo fissato, annullando così la 4 dimensione.





che l universo è una 3sfera e a me sembra un analogia funzionante..

Che sia una 3-sfera (spaziale, quindi) ad un istante fisso di tempo è vero ;).

Enrico Corsaro
26-05-2015, 06:48
Ragazzi mi complimento con tutti i partecipanti a queste belle discussioni...negli ultimi tempi mi sembra di aver visto un notevole aumento di attività in questi forum delle scienze astronomiche. Ovviamente mi fa molto piacere, continuate così :cool:.

DarknessLight
26-05-2015, 09:09
Giusto giusto certamente. Se ancora viene tenuto in vita significa che effettivamente va bene, ma ricorda, è sempre una approssimazione. Niente è perfetto qui, nè tantomeno l'omogeneità e l'isotropia.
Nulla toglie che in futuro il principio cosmologico possa essere rimpiazzato da una distribuzione effettiva della materia dentro l'Universo, in funzione delle 4 coordinate...per il momento ci teniamo il principio cosmologico.

ok. Ma tu semplicemente stai dicendomi che il principio cosmologico è un approssimazione della realtà. Ok chiaro.
provo a fare un esempio: se studio la distribuzione delle particelle all interno di un gas alla fine finisco per descrivere la loro densità e posizione secondo una stima statistica non reale ma approssimata che mi dice che le particelle del gas si distribuiscono in modo pressochè omogeneo tant è che possiamo considerarle tali. Questo sarebbe l equivalente del principio cosmologico, ovvero un approssimazione della distribuzione di materia.
un successivo studio più approfondito invece mi porta a definire meglio la distribuzione delle particelle nel gas dicendomi che non sono proprio perfettamente omogenee, bensì ci sono zone più dense e zone meno dense, ma statisticamente resta valida l omogeneità. Questo è l equivalente della distribuzione effettiva di materia in funzione delle 4 coordinate di cui parli alla fine del quote.
va bene l analogia? È semplicemente questo che mi vuoi dire? Ovvero che un giorno avremo una mappa precisissima della distribuzione di materia nell universo oppure vuoi dire qualcosa di più profondo che mi sfugge? Perché rinunciare al principio cosmologico mi SEMBRA di ritornare al vecchio conio dell osservatore preferenziale...Se la distribuzione non fosse omogenea allora neanche le forze lo sarebbero suppongo...



Capisco dove vuoi andare a parare, ma purtroppo non ha senso porsi la domanda in questi termini, nè cercare di immaginarsi una soluzione del genere, quando sappiamo già che le cose funzionano diversamente.
La risposta corretta è, l'universo è un oggetto in 4 dimensioni (che non possiamo visualizzare) e i raggi luminosi non torneranno mai al punto di prima anche in un tempo infinito, perchè l'Universo non è un oggetto statico, si evolve cioè in funzione del tempo, una delle quattro coordinate.

accantoniamo l analogia.
È giusto che il volume dell universo (la terza dimensione) è finito ma illimitato come una superficie chiusa?



Secondo me è fuorviante questa visione, abbi pazienza, perchè quello che fai è immaginare la coordinata temporale come una dimensione spaziale, quando invece non puoi visualizzarla. Io personalmente mi astengo dal prenderla per buona. Se poi la vuoi usare come esercizio mentale per immaginarti questo universo in 4 dimensioni OK, non c'è problema, purchè rimani consapevole del fatto che NON puoi visualizzarlo realmente. Puoi immaginare una 3-sfera spaziale, in uno spazio a 4 dimensioni spaziali, ed è una visione coerente, ma non in uno spazio-tempo a 4 dimensioni.
Quello che puoi fare è invece immaginare uno spazio a 3 dimensioni per un istante di tempo fissato, annullando così la 4 dimensione.

dimmi per favore come si fa a mettere il link di un articolo del forum così ti faccio vedere l articolo di Zappalà. Si tratta di una semplice immaginetta ma molto esplicativa.
io sinceramente trovo l analogia del cerchio (come la pone lui) ma anche quella della 3sfera molto esplicative.
è evidente che sono analogie e che la quarta dimensione per noi umani è concepita come temporale e non come spaziale, ma le analogie servono proprio per ricondurre i concetti a degli archetipi mentali più semplici da acquisire.
Assodato che non sono in grado di immaginare la quarta dimensione, io credo che ridurla ad una dimensione spaziale sia l analogia migliore.
come quando per descrivere il tempo si usa dire che il volume sia compresso in due dimensioni e il tempo diventa la terza dimensione (come Flatlandia).
L articolo di Zappalà fa lo stesso e io ho la necessità di capire come mai tu affermi che sia sbagliata.
se non fossi convinto non insisterei, più che altro è da tanto tempo che penso all universo in questo modo e devo essere sicuro di capire quale è stato il mio errore fino ad adesso...
Sai io come amatore dell arte grafica mi ritrovo bene con le analogie visive. Forse a te stringono perchè magari abbiamo approcci differenti alla cosa. Però certe cose mi sembrano troppo importanti per poterle sorvolare.



Che sia una 3-sfera (spaziale, quindi) ad un istante fisso di tempo è vero ;).

ok ;)
insomma tridimensionalmente è una superficie chiusa, finita, illimitata...

Enrico Corsaro
26-05-2015, 10:06
va bene l analogia? È semplicemente questo che mi vuoi dire? Ovvero che un giorno avremo una mappa precisissima della distribuzione di materia nell universo oppure vuoi dire qualcosa di più profondo che mi sfugge? Perché rinunciare al principio cosmologico mi SEMBRA di ritornare al vecchio conio dell osservatore preferenziale...Se la distribuzione non fosse omogenea allora neanche le forze lo sarebbero suppongo...

Dark l'analogia va bene, hai capito il discorso. Il principio cosmologico è una assunzione e come tale può essere soggetta a essere migliorata, non ci vedo nulla di strano in questo. Diciamo che va bene come approssimazione si, ma si sa che non è proprio così nella realtà, ed è un argomento in discussione nella cosmologia moderna (d'avanguardia diciamo). Non so se in futuro le cose cambieranno, ma certamente se dovesse decadere il principio cosmologico, la situazione diventerebbe molto più complicata.




dimmi per favore come si fa a mettere il link di un articolo del forum così ti faccio vedere l articolo di Zappalà. Si tratta di una semplice immaginetta ma molto esplicativa.

Semplicemente clicca sull'icona LINK (dove c'è il globo disegnato) e inseriscilo li.

DarknessLight
26-05-2015, 12:36
Ok, vediamo un po.
Il modo che ho sempre usato per pensare all universo è il seguente: penso ad una circonferenza come se rappresentasse il volume del cosmo e penso al raggio della circonferenza come se rappresentasse il tempo che trascorre, così che si parte da un punto adimensionale (ora senza soffermarci sulla veridicità o meno della singolarità) e via via immaginiamo di fare crescere il cerchio(quindi il raggio) simulando cosi l espansione dello spazio nel corso del tempo.
Così il raggio rappresenta il tempo trascorso in un determinato istante della storia dell universo, mentre il cerchio (la circonferenza) rappresenta il volume dell universo che si espande.

Ecco l articolo di Vincenzo Zappalà di cui ti parlavo ed in cui si cita questa analogia. Si tratta della prima immagine che trovi nell articolo http://www.astronomia.com/2013/01/03/sfere-cerchi-punti-veri-e-simbolici-alla-scoperta-dei-confini-delluniverso/

Io poi penso che si possa estendere l'analogia e dire ad esempio nel caso di una 2 sfera che il volume dell universo è rappresentato dalla superficie mentre la coordinata tempo coincide con il raggio (oppure giace sul raggio, é descritta dalla lunghezza del raggio, insomma ponila un pò come vuoi).

L ultima analogia è quella della 3sfera la cui "superficie" rappresenta un istante di universo, e il suo "raggio" è la coordinata temporale. Più aumenta il raggio più significa che è passato del tempo. E quindi la superficie tridimensionale della tre sfera aumenta.
Questa non è neanche un analogia perchè l universo è fatto proprio come una 3-sfera che si espande nella quarta dimensione.. almeno credo...

E qui ritorna l altra domanda: è giusto dire che il volume dell universo è una superficie finita ma illimitata? Io credo di sì perchè esso non ha bordi o confini e non si può uscire da esso..

Enrico Corsaro
03-06-2015, 20:58
Per evitare di fare confusione e portare alle lunghe questa discussione, che tra l'altro non è nemmeno di tema astrofisico ma cosmologico, ci spostiamo nel nuovo thread che hai aperto in cosmologia e vediamo di continuare li il discorso ok?
Prendiamo ovviamente di riferimento l'articolo che hai incluso qui ;).

DarknessLight
03-06-2015, 21:16
Per evitare di fare confusione e portare alle lunghe questa discussione, che tra l'altro non è nemmeno di tema astrofisico ma cosmologico, ci spostiamo nel nuovo thread che hai aperto in cosmologia e vediamo di continuare li il discorso ok?
Prendiamo ovviamente di riferimento l'articolo che hai incluso qui ;).

Certo certo. Era proprio questa la mia intenzione.
Comunque se guardi ho incluso l articolo di Zappalà anche nel nuovo thread ;)

Enrico Corsaro
03-06-2015, 21:52
Si avevo visto. Ho già provveduto ;).

Alex_71
03-08-2015, 07:35
Ciao, provo a risponderti.

L'esempio del palloncino, o della palla, è per farti capire che se tu sei ipoteticamente vincolato sulla superficie della sfera, e se la sfera si gonfia, vedrai allontanarsi tutto in ogni direzione sulla sua superficie da qualunque punto tu ti trovi. Lo spazio dell'Universo è una sfera a 3 dimensioni e per questo non è visualizzabile dall'esterno, come nel caso di un palloncino, ma l'analogia con il palloncino è perfettamente valida.

Hai quindi questa trasformazione
Superficie in 2D del palloncino --> Spazio in 3D dell'Universo

Come giustamente dici, questo spazio che aumenta al passare del tempo ha certamente delle proprietà interessanti. Esse sono principalmente legate alla geometria dello spazio-tempo.
Abbiamo scritto un articolo in merito (e credo ce ne siano anche altri nel portale) che ti consiglio di visionare.
Uno scritto da me lo puoi trovare QUI (http://www.astronomia.com/2015/04/22/il-modello-cosmologico-standard-%CE%BBcdm-parte-i-cose-e-come-si-ricava/).

Ciao,
scusate l'ignoranza, ma quando si utilizza la parola "espansione" vuol dire esattamente che la distanza fra due punti aumenta, oppure vuol dire che l'universo si sta stirando come un elastico, e quindi anche il mio ipotetico metro di misura si espande?
Spero di essermi spiegato :hm:

Red Hanuman
03-08-2015, 07:41
Ciao,
scusate l'ignoranza, ma quando si utilizza la parola "espansione" vuol dire esattamente che la distanza fra due punti aumenta, oppure vuol dire che l'universo si sta stirando come un elastico, e quindi anche il mio ipotetico metro di misura si espande?
Spero di essermi spiegato :hm:
Sì, a livello della distanza delle galassie si può apprezzare un'aumento della distanza tra di esse. Poi, per comodità possiamo utilizzare delle coordinate comoventi, in maniera da avere un riferimento fisso. Ma se parliamo di anni luce, allora il metro è sempre quello e la distanza aumenta.... ;)

DarknessLight
03-08-2015, 15:09
In effetti me lo sono sempre chiesto anch io.
Quando si parla di espansione dello spazio-tempo si intende che
1) si crea spazio-tempo dal nulla il che fa aumentare la distanza tra le stelle
oppure che
2) lo spazio-tempo si stira come un elastico o come la superficie del palloncino che si gonfia. La superficie del palloncino non aumenta, al massimo si stira e questo fa si che le stelle si allontanino, ma lo spazio-tempo non sta aumentando.

Sono due scenari molto diversi!

Quale dei due è quello giusto? La (1) o la (2)

Enrico Corsaro
03-08-2015, 15:16
In effetti me lo sono sempre chiesto anch io.
Quando si parla di espansione dello spazio-tempo si intende che
1) si crea spazio-tempo dal nulla il che fa aumentare la distanza tra le stelle
oppure che
2) lo spazio-tempo si stira come un elastico o come la superficie del palloncino che si gonfia. La superficie del palloncino non aumenta, al massimo si stira e questo fa si che le stelle si allontanino, ma lo spazio-tempo non sta aumentando.


Certamente la 1) per come le hai descritte ;).

DarknessLight
03-08-2015, 16:03
Certamente la 1) per come le hai descritte ;).

Ok
provo a fare un analogia per essere sicuro di aver capito.
Immaginiamo che lo spazio-tempo sia un metro da sarto.
nella (1) l espansione sarà rappresentata da un aggiunta di centimetri al metro da sarto, il che farà si che alla fine il metro sara' più grande. Così se all inizio avevo 100 centimetri ora ne ho 200 e questo rappresenta l espansione dello spazio-tempo: insomma, si crea spazio-tempo dal nulla.
nella (2) ( che a quanto pare è sbagliata) è come se NON si aggiungessero nuovi centimetri ma si stirassero. Ovvero se all inizio ho 100 centimetri anche alla fine ho sempre 100 centimetri, solo che ora 1 centimetro si è espanso e misura due centimetri. Quindi il risultato è il medesimo della (1), ma cambia il processo che porta a quel risultato.

Quindi se l analogia va bene, e se è la (1) quella corretta, allora significa proprio che lo spazio-tempo si crea dal nulla o sbaglio?

Enrico Corsaro
03-08-2015, 16:05
Si direi che hai afferrato il concetto. Lo spazio-tempo è un concetto astratto, non lo dimenticare, per cui non è vincolato a nessun principio di creazione, distruzione o trasformazione che invece regola i processi fisici veri e propri.

Marcos64
03-08-2015, 16:09
Grazie della precisazione, gia' mi stavo immaginando Lavoisier che si rivoltava nella tomba
come una trottola! :)